reproductive exammaster

Pataasin ang iyong marka sa homework at exams ngayon gamit ang Quizwiz!

Case A 15-year-old girl presents with short stature and primary amenorrhea; her neck is short and broad, her palate is high-arched, and her genitalia are infantile. Her parents recall several episodes of otitis media during childhood. Her serum FSH is elevated and estradiol is low. Question What would a chromosomal analysis most likely reveal? Answer Choices 1 46,XX 2 45,XO 3 47,XXY 4 47,XXX 5 46,XY

45,XO Explanation Turner syndrome (gonadal dysgenesis) is a genetic abnormality characterized mainly by karyotype 45,XO. Variants include 45,X/46,XX mosaicism and structural abnormalities of the X chromosome (i.e., rings and X-fragments). These patients are phenotypically girls/women with a variety of somatic stigmas (short stature, low hair line, widely-spaced nipples, webbed neck) and cardiac and renal malformations. Most patients have absent or incomplete pubertal development and primary amenorrhea; some may reach menarche spontaneously. When the diagnosis is made, a careful renal and cardiac examination should be performed to rule out malformations (horseshoe kidney, duplication of the collection system, coarctation of the aorta, bicuspid aortic valve, mitral valve prolapse). Treatment is directed towards estrogen replacement and management of other structural abnormalities. True hermaphroditism (46,XX/46,XY) is a rare chromosomal aberration; patients present with gradations between male and female external sexual organs (ambiguous genitalia) directed by the predominant gland. Internally, patients usually present with ovarian and testicular tissue in the same gonad (ovotestis), or an ovary on one side and a testis on the other side. Around 80% of patients with Klinefelter syndrome have the 47,XXY karyotype, and other aneuploid aberrations (e.g., 48,XXXY, 48,XXYY, and 46,XY/47,XXXY mosaicism) may be seen rarely. Patients with Klinefelter syndrome may present with small firm testes with associated low testosterone levels, high gonadotropin levels, gynecomastia, azoospermia, and increased leg length. Most patients fail to reach puberty and have small firm testes and signs of androgen deficiency. 47,XXX is a chromosomal aberration found in roughly 1 in 1000 female newborns. Somatic stigmata are absent in these patients, and clinical features are very non-specific (low birth weight, decreased head circumference). It is associated with varying degrees of intellectual disability and psychosocial adaptation problems. 46,XX constitutes the normal female karyotype.

Case A young couple presents for advice about infertility. They have been married for 4 years and have never used contraceptive methods. They are both 27 years old, and the wife has an 8-year-old daughter from a previous relationship. Both are very eager to have a child; the husband is somewhat anxious about the results of 2 semen analyses that show azoospermia. He states that when he was 14 years old he was evaluated for delayed pubertal development; at that time, he had a chromosomal analysis that revealed that the problem is genetic. He was prescribed testosterone, which he used for only a few months. He is 6'8'' tall and has unusually long legs. His facial skin is smooth; there is no beard. There is palpable breast tissue bilaterally. His right fifth finger is curved towards the fourth finger. His pubic hair is substantially decreased for his age, and both testes are small (approximately 5 cm) and firm in consistency. Question What is his karyotype likely to show? Answer Choices 1 46,XX/46,XY 2 45,X 3 47,XXY 4 46,XY/45,X 5 46,XX

47,XXY Explanation Around 80% of patients with Klinefelter syndrome are found to have the 47,XXY karyotype, although rarely other aneuploid aberrations such as 48,XXXY, 48,XXYY, and 46,XY/47,XXXY mosaicism may be seen. Patients with Klinefelter syndrome may present with small firm testes with associated low testosterone levels, high gonadotropin levels, gynecomastia, azoospermia, and increased leg length. The severity of phenotypic presentation depends on the number of X chromosomes or the presence of mosaicism. Fertility may be possible in patients with mosaicism. Signs of Klinefelter before puberty may be very mild and may go unnoticed; the majority of patients are diagnosed during evaluation for infertility. Most patients fail to reach puberty and have small firm testes and signs of androgen deficiency. Diagnosis is made by lymphocyte karyotyping; treatment includes testosterone replacement. In some patients, spermatozoa may be recovered by testicular extraction for intracytoplasmic sperm injection (ICSI). Most children born after ICSI will have a normal karyotype. True hermaphroditism (46,XX/46,XY) is a rare chromosomal aberration; patients present with gradations between male and female external sexual organs (ambiguous genitalia) directed by the predominant gland. Internally, patients usually present with ovarian and testicular tissue in the same gonad (ovotestis), or an ovary on one side and a testis on the other side. Diagnosis is made by the presence of ambiguous genitalia; most individuals are raised as boys. Although it is unknown if the ovarian tissue undergoes neoplastic degeneration, the risk of malignancy in the testicular tissue is increased and gonadectomy is usually recommended. Turner syndrome, also known as gonadal dysgenesis, is a genetic abnormality characterized mainly by karyotype 45,X. Other variants include 45,X/46,XX mosaicism and structural abnormalities of the X chromosome (e.g., rings, X-fragments). These patients are phenotypically girls/women with a variety of somatic stigmas (e.g., short stature, low hairline, widely spaced nipples, and webbed neck) and cardiac and renal malformations. Most patients have absent or incomplete pubertal development and primary amenorrhea, although a few may reach menarche spontaneously. When the diagnosis is made, a careful renal and cardiac examination should be performed to rule out malformations (e.g., horseshoe kidney, duplication of the collection system, coarctation of the aorta, bicuspid aortic valve, or mitral valve prolapse). Treatment is directed towards estrogen replacement and management of other structural abnormalities. 47,XXX is a chromosomal aberration found in roughly 1 in 1000 newborn girls. Somatic stigmata are absent in these patients and clinical features are very nonspecific (e.g., low birth weight, decreased head circumference). It is associated with varying degrees of intellectual disabilities and psychosocial adaptation problems. 46,XX constitutes the normal female karyotype.

Case A 27-year-old woman presents due to labial pain and burning and reportedly noticed a cluster of blisters in the area 2 days ago. She had myalgia, malaise, and a temperature of 100.5°F a few days prior to appearance of the blisters, which kept her home from work. She is monogamous and her partner denies similar symptoms. On physical examination, inguinal lymphadenopathy is noted in addition to the presence of multiple vesicles on the labia and perineum; some are ruptured and some are crusted. There is no vaginal discharge, and the rest of the pelvic exam is unremarkable. Question What is the proper treatment for this condition? Answer Choices 1 Penicillin 2 Acyclovir 3 Fluconazole 4 Metronidazole 5 Doxycycline

Acyclovir Explanation The symptoms described are typical for genital herpes, which is caused by the herpes simplex virus. There is no cure for herpes infections, but oral acyclovir, an antiviral drug, can reduce the number of recurrences. Penicillin would be treatment of choice for syphilis, caused by Treponema pallidum. Fluconazole, an antifungal drug, would be treatment of choice in candida vulva-vaginitis, caused by Candida species, mostly Candida albicans. Metronidazole, an antiprotozoal and antibiotic drug, is first-line therapy for bacterial vaginosis, which is caused by Gardnerella vaginalis; it is also the first-line therapy for trichomoniasis, which is caused by Trichomonas vaginalis. Doxycycline, a tetracycline, is often used in combination with other antibiotic drugs to treat pelvic inflammatory disease, which is usually caused by Chlamydia trachomatis and Neisseria gonorrhoeae.

Case A 25-year-old woman presents for her annual gynecological visit. You review her history and note menses onset at age 12 and duration of menses is typically around 6 days and occurs every 30 days. She is G0P0 and has no history of abnormal pap smears or diagnosed STIs. The patient is a non-smoker, is single, and is in a monogamous relationship with one partner for the past year. A pertinent positive the patient mentions is a whitish-gray vaginal discharge that increases after intercourse and is accompanied by a distinct musty odor; she denies any pain from this discharge. Physical examination and a positive result of a whiff test support your suspected diagnosis. Question What is the most likely diagnosis? Answer Choices 1 Candidiasis 2 Bacterial vaginosis 3 Trichomoniasis 4 Gonorrhea 5 Herpes simplex virus

Bacterial vaginosis Explanation This patient most likely has cervicitis caused by bacterial vaginosis. This is not viewed as a sexually transmitted disease. It is caused by overgrowth of Gardnerella and other anaerobes. These organisms produce a malodorous discharge with a characteristic grayish frothy appearance. An amine-like (fishy) odor is created if the discharge is alkalinized with 10% potassium hydroxide (positive whiff test). On wet mount, the characteristic clue cells will help confirm the diagnosis. A patient with a Candida infection will have a malodorous white curd-like discharge. It will also exhibit extreme pruritus and vulvovaginal erythema. A microscopic examination with potassium hydroxide will reveal both hyphae and spores. Trichomoniasis cervicitis (strawberry cervix) presents with diffuse vaginal erythema, red macular lesions, and punctate hemorrhages identified on the cervix. It produces a malodorous frothy yellow-green discharge and pruritus. Microscopic examination will reveal motile protozoan organisms with flagella. A patient who has a Gonorrhea infection will typically have signs and symptoms consistent with pelvic inflammatory disorder: lower abdominal pain, chills, fever, menses disturbances, purulent cervical discharge, and exquisite cervical and adnexal tenderness. This pathology is more common in nulliparous, young, sexually active women who have a history of multiple partners. Herpes simplex virus (HSV) will have distinct lesions involving the genital and anogenital area. Patients will experience burning and stinging and potentially have a neuralgia type of pain. The active lesions are usually tender small grouped vesicles and could potentially look eroded. Regional lymphadenopathy may also be present.

Case A 32-year-old woman presents with a 2-day history of having a vaginal "bump"; the bump is painful to sit on. She has never had this problem before, and she has been monogamous with the same sex partner for 7 years. On physical exam, you notice a solitary 2 cm smooth, slightly tender mass at the introitus. A KOH/wet mount demonstrates squamous cells with no white blood cells (WBCs), hyphae, or motile organisms. Whiff test is negative. Refer to the image. Question What is the most likely diagnosis? Answer Choices 1 Vaginal candidiasis 2 Herpes simplex, type 2 3 Lichen sclerosis 4 Bartholin gland cyst 5 Vaginal intraepithelial neoplasia (VAIN)

Bartholin gland cyst Explanation This patient most likely has a Bartholin gland cyst. The Bartholin glands are located at approximately the 4 o'clock and 8 o'clock positions at the introitus. Secretions help with vaginal lubrication. A cyst may form when the gland outlet is obstructed; infection occasionally develops, leading to an abscess. Both Bartholin cysts and abscesses can be painful. Initial conservative treatment consists of warm compresses to encourage drainage. If this is unsuccessful, incision and drainage are necessary. If Neisseria gonorrhea is implicated as a causative organism, appropriate antibiotic treatment should be prescribed. Vaginal candidiasis (also known as a yeast infection) would be characterized by symptoms of vaginal pruritus and discharge. Vulvar pain is common, and patients may note pain upon sitting, but a physical exam would reveal vaginal and vulvar erythema, thick white discharge, and hyphae on the wet mount. The fungal hyphae are especially noticeable when potassium hydroxide (KOH) is applied to the microscope slide to lyse the other cells. Vaginal candidiasis is not associated with a solitary vulvar mass. Herpes simplex, type 2 is typically the strain associated with genital herpes. A symptomatic patient with herpes may describe vulvar lesions causing pain with sitting. When lesions are visible, herpes is characterized by multiple painful vesicles and/or ulcerated lesions. Herpes lesions are typically much smaller than the described single mass. Lichen sclerosis is a chronic, progressive dermatological problem, often involving the vulvar and perineal epithelium. It is characterized by pruritus, dyspareunia, and fissuring of the skin. Lichen sclerosis is more common in post-menopausal women, and physical exam reveals thin, white-appearing tissue. Masses are not associated with lichen sclerosis. Vaginal intraepithelial neoplasia (VAIN) is typically located at the upper third of the vagina. Its presence is usually detected by an abnormality on the Pap pathology from asymptomatic patients. This patient's 2 cm mass at the introitus could potentially be a malignant tumor, but VAIN is an unlikely diagnosis. Bartholin gland carcinoma is a possibility, but it is very rare.

Case A 13-year-old boy presents with right breast development over the last 5-6 months. He is on the swim team, so this has caused him much consternation. The breast swelling is slightly tender and without any drainage. He has been healthy and denies taking any medications, he denies any substance abuse or trauma, and he is doing well at school. On exam, his height is 63 in (160 cm) and weight is 115 lb (53kg), which are both 75th percentile, BP is 94/68, pulse 68 beats/min. There is slightly tender 7-8 cm of right breast elevation and swelling extending from the areola that is non-erythematous. The left side is normal. There is no axillary lymphadenopathy. The testes are descended bilaterally and measure 3 cm in size. Pubic hair shows sparse growth of long downy hair at the base of the penis. There is no axillary or facial hair. He does have scant acne. The mother is very anxious and wants laboratory tests. Question What would be the most appropriate initial evaluation to rule out any underlying pathology? Answer Choices 1 Surgery referral 2 Breast ultrasound 3 Blood work 4 Urine drug screen 5 Nutrition consult

Blood work Explanation Approximately 70% of pubertal boys develop at least breast buds during adolescence. Some will have more extensive growth. The initial evaluation must include ordering blood work. This condition is probably a result of low testosterone relative to estradiol levels. Laboratory evaluation should include testosterone, estradiol, and gonadotropins to assess levels. This would effectively rule out conditions that could cause markedly abnormal levels, such as Klinefelter syndrome. The incidence of breast enlargement peaks at age 13-15 and may be unilateral or bilateral. Reassurance concerning the benign nature of the condition is usually all that is needed. Gynecomastia that occurs at other ages or stages of sexual development or that evolves rapidly would warrant further investigation. Physiologic gynecomastia usually resolves by age 18 when adult androgen/estrogen ratios are achieved. Surgical referral for removal of the enlarged breast glandular tissue should be considered in boys who have had very persistent gynecomastia and who have completed puberty. Ultrasound imaging of the breast can be used to distinguish between solid tumors and fluid-filled cysts. Breast cancer in adolescents is extremely rare. Tumors tend to be slow growing, well differentiated, firm, and painless. A urine drug screen would point to possible marijuana use that can be seen with gynecomastia when it is smoked heavily. There is little concern in this patient who is doing well at school, denies substance abuse, and whose parent does not express this concern. Obese pubertal boys commonly present with gynecomastia due to the accumulation of fat in the upper segment of the body. The breasts are often greater in size and more persistent than in usual physiologic gynecomastia. In such a case, a nutrition consult would be indicated, but this patient was seen to be proportional for both height and weight on exam.

Case A 26-year-old woman presents with 8 weeks of gastric reflux that does not improve with medication along with bloating, constipation, vaginal bleeding, and weight loss. Past medical history is significant for father with hypertension and coronary artery disease, mother with breast cancer, aunt with hypothyroidism, and sister with breast cancer. Complete physical examination is significant for an adnexal mass. CBC is significant for hemoglobin 11.5 g/dL and hematocrit of 38%. Urinalysis is negative. Question What would be the most appropriate tumor marker to order in this patient? Answer Choices 1 CA19-9 2 CEA 3 CA125 4 PSA 5 hCG

CA125 Explanation Cancer antigen 125 (CA125) is a tumor marker that can be used in the diagnosis of ovarian cancer. One test alone is not generally helpful because many things can cause an elevation in CA125, such as endometriosis, menstruation, pelvic inflammation, liver or kidney disease, and other types of cancers. Serial measurements are advised. The physical exam findings, family history of breast cancer, and symptoms of bloating, dyspepsia, constipation, and weight loss make the possibility of ovarian malignancy very high. While not all types of ovarian cancer will have elevated CA125, epithelial ovarian cancer (the most common type of ovarian cancer) will show elevated CA125 in most cases. According to some guidelines, women at high risk should discuss having CA125 and ultrasounds done yearly. The question specifically asks what tumor marker should be ordered in this patient (rather than what test would be ordered first). Given this patient's presentation of abdominal bloating, dyspepsia, constipation, and family history, the possibility of ovarian cancer is high; CA125 is the best answer. Cancer antigen 19-9 (CA19-9) is a marker used to detect pancreatic cancer. It can also be elevated in disease states such as biliary obstruction, cholangitis, liver cirrhosis, inflammatory bowel disease, and cystic fibrosis. Carcinoembryonic antigen (CEA) is most commonly associated with colon cancers, but it may be positive for vulvar tumors that originate in sweat glands. It is also positive in 35% of endometrial cancers and may be positive for ovarian cancer, but CA125 is more sensitive and specific for ovarian cancer than CEA. Prostate-specific antigen (PSA) may be positive in prostate tumors and is used to screen for this cancer in men. Human chorionic gonadotropin (hCG) is normally produced by the placenta. Elevated hCG levels are most commonly associated with pregnancy, but they can also be caused by choriocarcinoma of the uterus, embryonal carcinomas, and other germ cell cancers.

Case A 30-year-old woman presents with lower abdominal pain; she is thought to have pelvic inflammatory disease (PID). She admits to sex work and recreational drug use. Labs on admission reveal blood glucose of 260 mg/dL, a positive HIV screen, and a non-reactive RPR. Aside from the abdominal/pelvic pain, the admitting physical is notable for moderate obesity, absence of pronounced lymphadenopathy, and an erythematous macerated rash in the intertriginous distribution. The patient reports her rash is "really itchy and wet all the time," and it began within the last 3 months; it is now at its worst. She reports intense itching of the vulva over the last few weeks. Question The intertriginous rash is most likely a manifestation of what condition? Answer Choices 1 Atopic dermatitis 2 Candidiasis 3 Lichen planus 4 Psoriasis 5 Secondary syphilis

Candidiasis Explanation Candidiasis occurs due to overgrowth of yeast in areas of skin moisture: intertriginous zones of the groin and beneath pendulous breasts. Patients with diabetes, obesity, chronic intertrigo, and cellular immune deficiency are particularly susceptible. The rash often appears macerated and erythematous, with marked inflammation, satellite lesions, and scaling. Skin scrapings show pseudohyphae and yeast forms. The rash of secondary syphilis may also manifest as moist pink lesions (condylomata lata) in the intertriginous regions in 10% of patients. The rash, which may be subtle, usually manifests as symmetric mucocutaneous lesions with some truncal distribution. Non-tender generalized lymphadenopathy is usually present, and the primary chancre of syphilis will still be present in about 15% of patients. RPR should be positive. Psoriasis is a common chronic inflammatory condition of the skin. The lesions are variably pruritic and characterized by sharply demarcated papules and rounded plaques. A silvery scale is frequently observed covering the erythematous plaques. The most common psoriasis distribution is on the elbows, knees, sacral area/gluteal cleft, and scalp. The etiology is not well-defined. Atopic dermatitis is the skin's reaction to allergy (food, asthma, animal dander, etc.). In children and adolescents, it is frequently localized to the antecubital and popliteal fossae. Skin injury is more often a result of scratching than of the atopic process itself, and these individuals have a higher incidence of Staphylococcus aureus skin infections than unaffected patients. Lichen planus is a condition that produces primary lesions described as pruritic, polygonal, flat-topped, violaceous papules. Lesions may show thin gray lines (Wickham's striae) that tend to occur on wrists and shins, but they can occur anywhere on the skin. Mucous membranes, including the buccal mucosa, can be involved. The etiology is not completely understood, and the course is variable, but the lesions usually disappear spontaneously within several months to 2 years.

Case A 24-year-old woman presents with lower abdominal pain, nausea, and vomiting that has persisted for about 12 hours. She rates the pain at a 4 last night when it began, but she currently puts it at a 9; she states the pain seems to be worsening with each passing hour. She states she had her menses 1 week ago and noted an irregular flow and excessive vaginal discharge since it ceased. She is single. She admits to not being monogamous, and she only occasionally uses barrier contraception during sexual encounters. She takes a daily oral contraceptive pill. Question Considering the most likely diagnosis, what treatment regimen is most appropriate? Answer Choices 1 Ceftriaxone 250 mg IM and doxycycline 100 mg BID for 14 days 2 Metronidazole 500 mg PO BID for 14 days 3 Doxycycline 200 mg BID for 14 days 4 Metronidazole 500 mg PO BID for 14 days and doxycycline 100 mg BID for 14 days 5 Cefoxitin 2 g IM

Ceftriaxone 250 mg IM and doxycycline 100 mg BID for 14 days Explanation This patient most likely has an active case of salpingitis/endometritis, commonly referred to as pelvic inflammatory disease (PID). PID is a severe gynecological infection that is often polymicrobial; Neisseria gonorrhea and Chlamydia trachomatis are the most common sexually transmitted organisms that lead to PID. PID is frequently found in young nulliparous sexually active women with multiple partners. It is also considered the leading cause of infertility and ectopic pregnancy in this patient population group. PID presents with lower abdominal pain, chills, fever, menstrual disturbances, and purulent cervical discharge. Physical exam will reveal exquisite uterine, adnexal, or cervical motion tenderness and cervical discharge. Mild to moderate cases of PID may be treated on an outpatient basis. Recommended regimens include: single dose of cefoxitin 2 gm IM plus probenecid 1 gm PO along with doxycycline 100 mg PO BID for 14 days ceftriaxone 250 mg IM and doxycycline 100 mg BID for 14 days Metronidazole 500 mg PO BID for 14 days may be added to either of these regimens to help alleviate bacterial vaginosis, commonly associated with PID cases. Doxycycline 200 mg BID for 14 days will not provide coverage for Neisseria gonorrhea. Metronidazole 500 mg BID is the treatment of bacterial vaginosis and will not cover the most common organisms Neisseria gonorrhea or Chlamydia. Metronidazole 500 mg BID for 14 days and doxycycline 100 mg BID for 14 days will not provide coverage for Neisseria gonorrhea. A single dose of Cefoxitin 2 g IM will not provide coverage for Chlamydia.

Case A 34-year-old multiparous woman presents for a routine Pap smear after being "too busy" to have annual exams for the past 7 years. Three Pap smears in her 20s have all been normal. She has had one episode of venereal warts in her late teens; there was no recurrence. She had two vaginal deliveries. She does not smoke. Remainder of her history is negative. Her Pap smear is reported as "atypical squamous cells of undetermined significance." The HPV test is positive. Question What is the most appropriate next step in regards to evaluating the patient? Answer Choices 1 Colposcopy 2 Cone biopsy 3 Hysterectomy 4 Loop electrosurgical excision procedure (LEEP) 5 Repeat Pap in one year

Colposcopy Explanation Under the Bethesda system of reporting Pap smear results, the term atypia is reserved for abnormalities that are not clearly reactive in nature and do not the criteria for squamous intraepithelial lesions. Follow-up testing is required to make a definitive diagnosis. Colposcopy with directed biopsies of any identified abnormality is the appropriate next step in this patient because her Pap smear shows atypia and she is HPV positive. In a patient with a negative history for HPV disease, gets annual Pap smears, and who can be counted on for compliance, a follow-up Pap smear at a shorter interval (6 months) would be sufficient surveillance after a first atypical Pap smear. Cone biopsy and LEEP are not appropriate until a more specific tissue diagnosis is available. Hysterectomy is not indicated for an as yet undetermined cervical lesion.

A 24-year-old obese woman arrives at an infertility clinic accompanied by her husband. The couple has been married for 3 years, and they have been unsuccessful at conceiving a child. History and examination of the husband is unremarkable. She attained menarche at age 13 and gives history of irregular cycles for the past 5 years. There is no history of pelvic pain or discomfort. The patient's last menstrual period was 2 months ago. On physical exam, the woman has hirsutism and acne, and her pelvic examination is unremarkable. Investigations reveal elevated serum levels of dehydroepiandrosterone sulfate (DHEAS) and androstenedione. Question What is the most likely cause of infertility in this patient? 1 Polycystic ovarian syndrome 2 Pelvic inflammatory disease 3 Endometriosis 4 Leiomyoma 5 Endometrial carcinoma

Correct answer: Polycystic ovarian syndrome Explanation The clinical presentation and investigatory findings are suggestive of polycystic ovarian syndrome, characterized by infertility, obesity, hirsutism, and menstrual disturbances; irregular menses can sometimes be seen rather than amenorrhea. The ovaries can have numerous cysts. Diagnosis can be made on clinical and biochemical grounds of hyperandrogenism and oligo-ovulation after exclusion of other disorders such as congenital adrenal hyperplasia and hyperprolactinemia. Other features observed are overt diabetes due to insulin resistance, acanthosis nigricans, hypertension, and dyslipidemia. Ultrasound examination may reveal ovaries studded with multiple tiny follicular cysts surrounding the ovarian stroma. The aim of treatment is suppression of androgen production by the use of oral contraceptives or medroxyprogesterone acetate. Clomiphene citrate or gonadotropins are used for those desiring conception. Pelvic inflammatory disease (PID) can also lead to infertility, but there would not be an association of PID with obesity, amenorrhea, or with features of hyperandrogenism such as hirsutism, acne, and elevation of DHEAS and androstenedione. Dysmenorrhea can be seen with PID. A patient with PID would usually present with lower quadrant pain. On physical exam, positive findings are expected. Endometriosis can present with dysmenorrhea and irregular menstrual cycles, but there is no association of endometriosis with obesity, hirsutism, and other features of hyperandrogenism. Leiomyomas are usually asymptomatic. Obesity and hirsutism are not associated with leiomyomas. Menorrhagia is a more frequent presentation with leiomyomas. Endometrial carcinoma is typically seen in postmenopausal women. Obesity is a risk factor for endometrial carcinoma.

Case You are performing an annual physical examination of a 14-year-old girl. Over the last 3 years, she has been treated for the depression related to her parents' divorce process. She complains of frequent constipation followed by loose stools and is treated for dermatitis herpetiformis. She participates in swimming competitions and spends about 18-20 hours per week training. However, her mother is worried because she has not grown enough and still has not gotten her period (her mother had her period when she was 12 years old). The rest of her personal and family history is not contributing. Your patient's BMI is 15 (percentile 3%); she is in Tanner stage 2 (the same as last year); and her bone age is 12.5 years. The rest of the physical examination is normal, and complete blood count results are within normal limits. Question What is the most likely diagnosis? Answer Choices 1 Major depression 2 Anorexia nervosa 3 Constitutional development delay 4 Hypergonadotropic hypogonadism 5 Hypogonadotropic hypogonadism

Correct answer: Hypogonadotropic hypogonadism Explanation This patient has a delay in puberty, in weight gain, and in linear growth, alongside gastrointestinal symptoms. That should prompt you to think about the systemic disease with chronic undernutrition (in this case, most likely celiac disease). Her hypothalamic-pituitary-gonadal axis also may be suppressed because of the stress and the excessive exercise. She most likely has hypogonadotropic hypogonadism. Hypogonadotropic hypogonadism may present prior to or after the completion of puberty. It involves slowed gonadotropin-releasing hormone release, leading to low levels of FSH and LH. Your patient was under stress, but she also has bone maturation and growth and weight gain delay, which should be investigated. Constitutional development delay is a diagnosis of exclusion. Children with constitutional delay typically present with delayed growth, adrenarche, and sexual development associated with a decline in growth velocity and delayed skeletal maturation. You must exclude other causes before making the diagnosis of the constitutional delay. In this case, you have signs and symptoms that make constitutional delay unlikely. Hypergonadotropic hypogonadism is due to the impaired response of the gonads to the gonadotropins. In most cases, it is congenital (chromosomal aberrations, enzyme defects), is part of different syndromes, and usually is not accompanied by a delay in growth, weight, and anemia. Low body weight could be found in patients with major depression and anorexia nervosa but additional findings of delayed tanner staging and reduced bone age require hormonal investigation to rule out other diagnoses.

Case A 26-year-old woman is in the 24th week of her first pregnancy. She is in fairly good shape, and the pregnancy is progressing well, but a fasting blood glucose done in the office shows it to be 146 mg/dL. It is repeated the next day, and the value is 142 mg/dL. An oral glucose tolerance test is ordered, which comes back as abnormal. She is diagnosed with gestational diabetes and advised to meet with her obstetrician. Question What should be the recommended regimen for this patient? Answer Choices 1 Diet and liraglutide if blood glucose remains high despite diet control 2 Diet and metformin if blood glucose remains high despite diet control 3 Diet and sulfonylurea if blood glucose remains high despite diet control 4 Diet and insulin if blood glucose remains high despite diet control 5 Diet and acarbose if blood glucose remains high despite diet control

Diet and insulin if blood glucose remains high despite diet control Explanation Diet and insulin if blood glucose remains high despite diet control is the correct response. Gestational diabetes mellitus (GDM) should be treated early and optimally to reduce risks to the fetus. In pregnancy, there is an increased resistance to insulin, which causes high levels of glucose, lipids, and insulin. Placental lactogen and high levels of estrogen and progesterone cause this scenario. Risk factors for GDM include family history of diabetes, prior delivery of large baby, age over 25, obesity, steroid use, glycosuria in first antenatal visit, ethnicity like Hispanic, African, Native American, Asian or Pacific Islander, polycystic ovarian disease (PCOS), and prior fetal loss of unknown cause. Universal screening of all pregnant women is recommended (though very low-risk women may be excluded), usually at 24-28 weeks of pregnancy, except if risk factors are evident (e.g., obesity, prior history of GDM), in which case screening should be done as early as possible. A random blood sugar of >200 mg/dL or fasting glucose of >126 mg/dL on 2 occasions is diagnostic of diabetes and does not require screening. Screening is done with a 50 gm oral glucose challenge test. A value of >140 mg/dL is positive, and a 3-hour glucose tolerance test is then done for definitive diagnosis. GDM is confirmed if 2 or more of the following are present: fasting glucose> 95 mg/dL, 1-hour glucose >180 mg/dL, 2-hour glucose >155 mg/dL, and 3-hour glucose >140 mg/dL. Referral to a nutritionist for strict diet control is a must to maintain euglycemia, prevent ketosis, and monitor adequate weight gain. Blood glucose should be measured before breakfast and 1 hour after each meal by the patient. Moderate exercise is encouraged. If blood glucose remains high despite these measures, then insulin is the treatment of choice. In the United States, as of 2018, insulin is still recommended as first-line therapy by the American College of Obstetricians and Gynecologists, as well as by the American Diabetes Association. The use of oral antihyperglycemic agents, including metformin, sulfonylurea, and acarbose, are permissible in pregnancy, but due to a comparative lack of evidence, these are typically reserved for those patients who refuse (or cannot afford) insulin therapy. These oral hypoglycemic agents are not approved for use in pregnancy by the FDA in the USA at this time and are not the initial first-line choice for treatment. Liraglutide is contraindicated during pregnancy due to lack of potential benefit and possible fetal harm.

Case A 20-year-old Lebanese woman wants to start birth control. She has never been sexually active, and her wedding is in 2 months. She feels well and reports no issues. She thinks she wants "the pill." Her fiancé is also a virgin, and they are not interested in condoms or other barrier contraception. She wants to delay childbearing for at least 2 years. Past medical history includes occasional OTC ibuprofen for menstrual cramps and headaches, penicillin allergy, previous tonsillectomy, menarche age 12, and regular menses with mild-moderate dysmenorrhea. Her older sister had a blood clot in her lung after giving birth. Her paternal grandfather has diabetes and hypertension, maternal grandmother had a stroke, mother had a DVT in a leg, and maternal grandfather had prostate cancer. She denies tobacco, alcohol, and recreational drug use. Vitals: 123 lb, 5'4", BMI 21.1, pulse 88, BP 134/86, temperature 97.9°F. Question What test should be ordered before prescribing birth control pills? Answer Choices 1 Bleeding time 2 Complete blood count (CBC) 3 Complete metabolic panel (CMP) 4 Factor V Leiden 5 Pap smear

Factor V Leiden Explanation Screening tests are not usually necessary before starting most patients on contraceptives, but Factor V Leiden (especially homozygous FVL) substantially increases risks of potentially life-threatening VTE and affects the ability to carry a baby to term. This patient is healthy, but her family history is concerning for a heritable form of hypercoagulability (FVL). She should be tested for a Factor V Leiden (FVL) mutation, estimated in approximately 5% of the Caucasian population and highest in those of Lebanese ethnicity (prevalence >14%). Patients who test positive should not use estrogens in contraceptives or menopausal hormone therapy due to increased risk of thromboembolism. Bleeding time is a test of platelet function, not coagulation factors. Bleeding time is useful in patients at risk of bleeding too much. Complete blood count (CBC) can be normal in Factor V Leiden mutation. Familial or essential thrombocytosis is another condition of inherited hypercoagulability that would show up on the CBC, but it is rare and often presents at birth; this patient is unlikely to have it. Complete metabolic panel (CMP) does not provide information related to this patient's family history of hypercoagulability. Many hormonal contraceptives are hepatically metabolized, so CMP could theoretically ensure normal liver enzymes, but routine testing of liver function is not recommended before starting birth control. There is no compelling reason to obtain a Pap smear to initiate birth control. Routine cervical cancer screening guidelines indicate that this patient should wait until age 21 before beginning Pap testing.

Case A 22-year-old woman presents due to feeling depressed, withdrawn, and irritable from 3 days prior to her menses until the day after her flow begins. She frequently misses her college classes, and she is concerned about her symptoms' potential impact on her academic performance. She is otherwise in a good state of health, and she has no history of chronic medical or psychiatric disorders. She is not taking any medication. She does not use tobacco, drink alcohol, or use illicit drugs. Menarche was at age 13, and her menses are currently regular each month. She denies cramping, bloating, and other associated physical symptoms. She is sexually active with one partner, and she uses condoms for contraception. Question Following prospective documentation and confirmation of her symptom pattern, what would be the single best first-line treatment for this condition? Answer Choices 1 Alprazolam 2 Calcium carbonate 3 Fluoxetine 4 Leuprolide 5 Norgestimate plus ethinyl estradiol

Fluoxetine Explanation The clinical scenario presented here is describing premenstrual syndrome (PMS). PMS and premenstrual dysphoric disorder (PMDD) constitute a group of disorders and symptoms related to the menstrual cycle. At least four criteria must be met in order to make the diagnosis. Symptoms must be cyclic and have a consistent and predictable relationship to the luteal phase of the menstrual cycle, and they must be relieved within 4 days of the onset of menses be sufficiently severe to interfere with some aspects of life not be better explained by another diagnosis be present without drug, alcohol, or hormone use The physiologic and the psychosocial aspects of PMS as well as the severity of the presentation must be considered when designing a therapeutic program. All treatments regimens for PMS/PMDD are ultimately aimed at alleviating symptoms and improving occupational and psychosocial functioning, leading to an enhanced quality of life. This patient has classic PPMD with mood symptoms (feeling depressed, withdrawn, and irritable) predominating the clinical picture. The condition is affecting her school performance. PPMD is considered the most severe form of PMS and is best managed by pharmacologic intervention with SSRIs such as Fluoxetine, as the serotoninergic antidepressants are currently recognized as first-line treatment of choice for PMDD with mood symptoms predominating. Calcium carbonate has been shown to be an effective non-pharmacologic option in the treatment of PMS. In this case, however, the patient appears to be presenting with classic PMDD with mood symptoms impairing her occupational performance, so the best first-line treatment is pharmacologic intervention with SSRIs. Alprazolam is a benzodiazepine that has been reported to be beneficial in PMS, but it is not a first-line therapy due to its addictive potential. Hormonal contraception (norgestimate plus ethinyl estradiol) is indicated for PMS physical symptoms, and it is most effective when taken continuously. These agents should not be used if mood symptoms are primary. Since she denies physical symptoms, they would not be appropriate in this patient. Leuprolide, an agonist at pituitary GnRH receptor, would suppress menstrual cycles; it is used as a treatment for PMS, but it is associated with significant side effects.

Case A 48-year-old Caucasian woman presents due to feeling like she is losing her mind. She wants some tests done. Upon further questioning, she reports she is having multiple episodes daily in which she suddenly becomes very hot, flushed, and diaphoretic. These episodes last about 1 minute, then resolve. She has not measured a fever. The patient reports that the episodes occur during the day and at night, causing her to wake up drenched in sweat. As a result, her sleep has been poor, and she feels fatigued and irritable at both work and home. She has noticed these symptoms for about 2 months, and they seem to be increasing in severity. The patient has not had a period for 3 months; she recently did a home pregnancy test, which was negative. Prior to that, she had regular menses. This patient denies weight changes, palpitations, cold intolerance, bowel changes, as well as changes in her nails, skin, and hair. Although she admits irritability, she denies anxiety, depressed mood, and suicidal ideation. Her family history is remarkable for diabetes in her maternal grandfather and hypertension in her father. She is a G4P3Ab1. She denies any major psychosocial stressors recently. She drinks alcohol rarely, and she denies use of other drugs. Vitals and a urine specimen for hCG were obtained prior to the physical exam. Weight168 lbHeight65"Pulse72Blood pressure120/82Temperature98.2°FUrine hCGNegative Complete screening physical exam is normal, with normal sexual development and absence of hirsutism and acne. Question What diagnostic study result is most consistent with the suspected diagnosis? Answer Choices 1 Decreased free thyroxine (free T4) 2 Decreased luteinizing hormone (LH) 3 Increased estradiol 4 Increased follicle stimulating hormone (FSH) 5 Increased testosterone

Increased follicle stimulating hormone (FSH) Explanation This patient would be expected to have increased follicle stimulating hormone (FSH) levels. Based on the history and physical, her diagnosis is perimenopause, with a classic history of vasomotor symptoms (hot flashes/flushes and night sweats). Perimenopause is defined as the time prior to menopause in which the woman's ovarian hormone output begins to decline, often beginning in the mid-40s. Menopause is defined at the point in time in which the patient has been amenorrheic for 12 months. Postmenopause occurs after that. The average age for menopause is just over 51 years of age. With her history, a negative pregnancy test, and normal physical exam at age 48, the diagnosis of perimenopause can actually be made without diagnostic studies. If tests are done, however, it is recommended to check the "upstream" hormones from the pituitary (follicle-stimulating hormone and/or luteinizing hormone) rather than the ovarian hormones themselves (estradiol and progesterone). FSH elevation is the first measurable hormone evidence of menopausal ovarian failure. With the physiologic feedback mechanism, both FSH and LH rise in response to a low ovarian output of inhibin, which would normally suppress FSH in a younger woman. A decreased free thyroxine (free T4) level would indicate hypothyroidism. Although thyroid function should be considered in women having menstrual changes and fatigue, this patient denies other symptoms of hypothyroidism (weight gain, dermatologic changes, cold intolerance, constipation, and depressed mood). Hypothyroidism is not associated with hot flashes and night sweats. Decreased luteinizing hormone (LH) would be expected in situations in which there is primary pituitary or hypothalamic dysfunction or suppression, such as with the administration of hormonal contraceptives. Eating disorders and malnutrition may also cause low LH levels. This patient presentation does not indicate any of these disorders. In perimenopause, and more dramatically in menopause, estradiol levels decrease. An increased estradiol level would not be expected in this patient. Increased testosterone levels are most often seen in women due to polycystic ovarian syndrome (PCOS); they can also be seen in women with primary tumors of the ovaries, in which androgen production is increased. PCOS is characterized by menstrual changes consistent with anovulation (amenorrhea, oligomenorrhea, and history of infertility), hyperandrogenism (e.g., hirsutism and acne), and characteristic polycystic ovaries are seen on ultrasound. PCOS may be associated with obesity and insulin resistance or diabetes. The ovarian tumors are rare. This patient does not have a presentation consistent with either PCOS or ovarian tumor.

Case A 50-year-old woman presents for her annual pelvic examination. She states her last menstrual period was over 12 months ago; the last few occurrences of menses were extremely irregular. The patient also describes having the sensation of intense heat in her face and trunk; the sensation is accompanied by sweating. She further states that these "heat episodes" have been occurring 1 or 2 times a week for the last several months. She reports no other issues at this time. She has received her annual pap and pelvic examination yearly, as well as a clinical breast exam, without any issues. During the pelvic examination, you note obvious vaginal thinning and excessive dryness; there is also apparent vaginal wall atrophy. Question Laboratory findings in this patient would include a decreased serum estradiol and what else? Answer Choices 1 Decreased serum thyroid-stimulating hormone (TSH) 2 Increased serum follicle-stimulating hormone (FSH) 3 Decreased serum luteinizing hormone (LH) 4 Increased serum prolactin level 5 Decreased serum aldosterone level

Increased serum follicle-stimulating hormone (FSH) Explanation This patient scenario is most likely caused by menopause. Menopause is essentially a cessation of menstruation from either natural aging (usually amenorrhea for at least 12 months) or an external cause (surgical). There is usually a 1- to 3-year period during which women will typically adjust physiologically to the diminished hormonal and menstrual actions and the effects this has on their body. These effects may include hot flashes, night sweats, vaginal dryness, and in the later stages, osteoporosis. The average age of menopause in western societies is around 51. Laboratory findings that indicate natural menopause include elevated levels of follicle-stimulating hormone (FSH) and luteinizing hormone (LH), decreased levels of estradiol (estrogen), and normal levels of TSH and prolactin. Decreased levels of TSH would be seen in hyperthyroidism (thyrotoxicosis). Menstrual irregularities are seen with hyperthyroidism, but menstrual regularity will most likely return to its normal state if TSH levels are corrected. Serum prolactin levels will remain unchanged in naturally occurring menopause. Levels may be increased if amenorrhea is a consequence of a prolactin-secreting pituitary adenoma. Decreased levels of aldosterone in relation to amenorrhea may be found in patients who are experiencing toxemia of pregnancy, which is not likely in this patient.

Case A 31-year-old woman presents with her husband to discuss methods of temporary contraception. They explain that they want to have children in the future but would like to wait a few years. They are both in good health. The woman's routine pelvic and physical examinations show a healthy young woman; her past medical history is unremarkable. She is a non-smoker and there is no family history of breast, ovarian, or uterine cancer. Question What would be the most effective means of temporary contraception for this couple? Answer Choices 1 Morning-after pill 2 Condoms 3 Diaphragm 4 Intrauterine device 5 Cervical cap

Intrauterine device Explanation Contraceptives have both a theoretical effectiveness, which is a lab-tested value, and an actual effectiveness, which is how well they perform when used by actual people. In terms of contraceptive methods, intrauterine devices have a 99% theoretical effectiveness, and about a 97% actual effectiveness, making it one of the most effective methods of birth control available. Other benefits include its temporary nature and minimal side effects. Other widely used methods for fertility control include barrier methods. These include diaphragms, sponges, condoms, and the cervical cap. Although the theoretical effectiveness of these methods is high (about 94-98%), their actual effectiveness is probably somewhere in the high 80s due to user error. The morning-after pill (Plan B) is used after unprotected intercourse, but it has been shown to not be as statistically effective as the oral contraceptive pill taken as directed. It is also not intended for repeated use.

Case A 20-year-old primigravida presents at 30 weeks gestation with a 2-day history of a headache, decreased urine output, and facial puffiness. On examination, vitals are as follows: pulse 90/min, blood pressure 164/116 mm Hg and 166/114 mm Hg (taken 6 hours apart), RR 20/min. There is generalized edema and exaggerated deep tendon reflexes with presence of clonus. Abdominal examination reveals a fundal height corresponding to 30 weeks gestation and the presence of good fetal heart sounds (FHS). Urine dipstick for protein reveals 3+proteinuria (300 mg/dL) on 2 occasions. Question What is the next step in the management of this patient? 1 Strict bed rest at home 2 Intravenous phenytoin 3 Immediate induction of labor 4 Immediate cesarean section 5 Injection of betamethasone

Injection of betamethasone Explanation Preeclampsia is a pregnancy-induced multi-system disorder characterized by development of hypertension (>140/90) and proteinuria after 20 weeks gestation. Severe preeclampsia is characterized by systolic pressure ≥160 mm Hg and a diastolic pressure ≥110 mm Hg measured on two occasions at least 6 hours apart. Proteinuria often occurs and is characterized by excretion of >5 g/day or dipstick urine testing showing 3+ or higher proteinuria on two occasions. Other signs and symptoms include headache, visual disturbances, abdominal pain, oliguria, pulmonary edema, signs of clonus, impaired liver function, thrombocytopenia, and intrauterine growth restriction. The only cure for preeclampsia is delivery of the fetus and placenta. Neonates delivered prematurely are at risk for breathing problems because their lungs may not be fully developed. Women who require pre-term delivery (<34 weeks) as described in this case are usually given two injections of corticosteroid (e.g., betamethasone) 24 hours apart to speed lung development. Full benefit occurs 48 hours after the first injection. Steroid treatment also decreases other potential complications of preterm delivery, such as intraventricular brain hemorrhage. This patient with severe preeclampsia must be carefully assessed for signs of imminent eclampsia and continual monitoring in a hospital setting is required. For the prevention of seizures, magnesium sulfate is the preferred drug, not intravenous phenytoin. Induction of labor should not take place until after 48 hours of corticosteroid treatment to promote lung maturation for delivery. Women with preeclampsia can develop seizures (eclampsia) and are usually treated with magnesium sulfate during labor and 24 hours after delivery to prevent seizures. Immediate cesarean section, if required, should not take place until after completion of 48 hours of steroid treatment to enhance lung maturation prior to delivery in women at <34 weeks of gestation with preeclampsia.

Case A 16-year-old girl presents to her primary care physician after having discovered a breast mass while bathing. She reports no symptoms of any kind; the mass was discovered while taking a shower. There is no history of chest trauma. Exam reveals Tanner stage IV breast development, appropriate to age, and a 2 cm mass in the upper outer quadrant of the right breast. The mass is rubbery in character, mobile, with distinctly palpable borders, non-tender, not fixed to adjacent tissue, and without change in surface anatomy of the breast. Question What is the most likely diagnosis? Answer Choices 1 Breast abscess 2 Phyllodes tumor 3 Juvenile fibroadenoma 4 Giant fibroadenoma 5 Malignancy

Juvenile fibroadenoma Explanation A juvenile fibroadenoma is the most common breast mass in teens. Histology involves stromal proliferation surrounded by compressed or distorted ducts. Most occur in teens age 14 or over. Exam reveals a firm non-tender rubbery mass that is mobile and has well-demarcated margins. Average size is 2-3 cm. Two-thirds will be in the upper outer quadrant. The diagnosis is made by clinical exam, fine needle aspiration, core biopsy, or excisional biopsy. Treatment is surgical excision. A giant fibroadenoma displays more rapid growth, more stromal cellularity, and larger size (>5 cm). Although encapsulated, a giant fibroadenoma has the consistency of normal breast tissue, so the margins may be difficult to define. Surface anatomy changes include a sensation of warmth over the mass due to increased blood supply and dilation of superficial veins. Phyllodes tumors are of low prevalence. Affected teens present with a large bulky breast mass up to 20 cm. The mass is firm and mobile and may be smooth or irregular. Its large size causes stretching of overlying skin with dilation of superficial veins. Skin retraction, nipple retraction, and nipple discharge may be present. The mass lacks a capsule, and it may extend into surrounding breast tissue. Biopsy establishes the diagnosis. A breast abscess is caused by bacteria introduced from the skin into ductal tissue. It is not common in adolescents. The most common pathogen is Staphylococcus aureus. Presentation is similar to that of any other abscess: redness, warmth, swelling, and pain. Breast malignancy is reported only sporadically in girls/women under 20. One-third are primary malignancies of breast tissue; the remainder arise from non-breast tissue or represent metastases from other primary sites. The breast mass is usually hard, sub-areolar, fixed to deep tissue, and of varying size.

Case A 22-year-old woman presents for her annual well-woman examination. She was seen last year and she had her first pap smear done, which showed no evidence of intraepithelial neoplasia. She is sexually active with one partner currently and a history of two partners total. She reports using condoms intermittently. Question According to current guidelines, what type of cervical cancer screening should be done on this patient at this visit, if any? Answer Choices 1 Pap smear with reflex high-risk HPV testing 2 Pap smear and high-risk HPV testing 3 Pap smear alone 4 High-risk HPV testing alone 5 No cervical cancer screening recommended Show Explanation

No cervical cancer screening recommended Explanation According to the 2012 ASCCP (American Society for Colposcopy and Cervical Pathology) guidelines, women ages 21-24 should be screened conservatively for cervical cancer. Following a normal cervical cancer screening result, this screening could consist of a pap smear alone or a pap smear with reflex high-risk HPV testing every 3 years. According to these guidelines, the patient in the scenario should have her next screening done at age 24.

Case A 45-year-old Caucasian woman presents with a 2-month history of a lump in her right breast. She has had no children. Her menstrual periods began at age 15 and are still regular. She is a successful businessperson who does not smoke cigarettes and drinks 6 cups of coffee per day. She gets 30 minutes of physical activity at least 5 times weekly. On examination, she has a nontender lump in the upper outer quadrant of her right breast. Several matted right axillary lymph nodes are also palpable. Question Given her history, what is her greatest risk factor for developing breast cancer? 1 Regular physical activity 2 Menarche at 15 years old 3 Menopause at age 45 years old 4 Excessive caffeine consumption 5 Nulliparity

NulliparityExplanation The correct response is nulliparity. The clinical features are suggestive of breast cancer. Although it occurs mainly in women, a small percentage develops in men.Breast cancer has no single cause, but multiple risk factors have been identified. These include: Family history of premenopausal breast cancer in a first-degree relative (e.g., mother, daughter, sister) Having the first full-term pregnancy after age 30 Prior personal history of breast cancer Chest exposure to high dose of ionizing radiation Nulliparity Early menarche at or before age 12 Late menopause after age 50 Taking oral contraceptives (birth control pills) Lack of physical activity Current or recent use of oral contraceptive pills (for women who stopped using oral contraceptive pills 10+ years ago, there is no apparent increase in risk).

Case A 48-year-old Caucasian woman presents due to feeling like she is losing her mind. She wants some tests done. Upon further questioning, she reports she is having multiple episodes daily in which she suddenly becomes very hot, flushed, and diaphoretic. These episodes last about 1 minute before resolving. She has not measured a fever. The patient reports that these episodes occur during the day and at night, during which she awakes drenched with sweat. As a result, her sleep has been poor, and she feels fatigued and irritable at both work and home. She has noticed these symptoms for about the last 2 months, and they seem to be increasing in severity. The patient has not had a period for 3 months; she recently did a home pregnancy test, which was negative. Prior to that, she had regular menses. This patient denies weight changes, palpitations, cold intolerance, bowel changes, as well as changes in her nails, skin, and hair. Although she admits irritability, she denies anxiety, depressed mood, and suicidal ideation. Her family history is remarkable for diabetes in her maternal grandfather and hypertension in her father. She is a G4P3Ab1. She denies any major psychosocial stressors recently. She drinks alcohol rarely, and she denies use of other drugs. Vitals and a urine specimen for hCG were obtained prior to the physical exam. Weight168 lbHeight65"Pulse72Blood pressure120/82 mm HgTemperature98.2°FUrine hCGNegative Complete screening physical exam is normal, with normal sexual development and absence of hirsutism and acne. Question Based on this patient's history and physical, what is the most reasonable management for this patient? Answer Choices 1 Offer blood tests and provide education/prescription for hormone therapy. 2 Offer blood tests and provide education/prescription for raloxifene (Evista). 3 Order cancer antigen 125 (CA-125). 4 Order pelvic ultrasound. 5 Order thyroid ultrasound.

Offer blood tests and provide education/prescription for hormone therapy. Explanation Based on the history and physical, her diagnosis is perimenopause, with a classic history of vasomotor symptoms (hot flashes/flushes and night sweats). Perimenopause is defined as the time prior to menopause in which the woman's ovarian hormone output begins to decline, often beginning in the mid-40s. Menopause is defined at the point in time in which the patient has been amenorrheic for 12 months. Postmenopause occurs after that. The average age for menopause is just over 51. With a negative pregnancy test and normal physical exam at age 48 with this history, the diagnosis of perimenopause can actually be made without diagnostic studies. If tests were done, an elevated follicle-stimulating hormone is the best indicator of declining ovarian function. The most effective treatment for vasomotor symptoms is hormone therapy, and the patient's history gives no contraindications for its use. Because this patient presents to the clinic desiring tests, the clinician should offer blood tests and provide education/a prescription for hormone therapy. Raloxifene (Evista) is a selective estrogen receptor modulator (SERM), which is used in preventing and treating osteoporosis and prevention of breast cancer. It is not useful for vasomotor symptoms, and increased hot flashes are a common side effect of this medication. This would not be an appropriate medication choice for this patient. As mentioned above, blood tests could be offered but are not necessary. The cancer antigen 125 (CA-125) is a marker primarily associated with ovarian cancer. It is most useful in monitoring patients with a history of ovarian cancer for recurrence and in the evaluation of suspicious solid ovarian masses. This patient's presentation does not support a solid reason for ordering a CA-125. A pelvic ultrasound is useful in evaluating many gynecologic anatomic abnormalities, but it will not demonstrate function. In this patient's case, she presents with declining ovarian function due to perimenopause, and this is undetectable on ultrasound. Her physical exam was normal. If she had presented with pelvic complaints, an abnormal pelvic exam, or irregular or heavy menstrual bleeding, an ultrasound could have been useful. Although thyroid disorders can cause menstrual cycle changes and fatigue, ordering a thyroid ultrasound is not indicated in this case. A more appropriate way to rule in or rule out thyroid disease is with appropriate history, physical exam, and a serum thyroid-stimulating hormone level. This patient denies other symptoms of hypothyroidism (weight gain, dermatologic changes, cold intolerance, constipation, and depressed mood). Hypothyroidism is not associated with hot flashes and night sweats.

Case A 51-year-old Caucasian woman presents to the clinic to follow up on multiple tests she had requested from another practitioner. She has had what she describes as "episodes," in which she feels overheated and diaphoretic. Her coworkers note that her face turns red. These episodes seem to occur multiple times during the day; they have been occurring for the last 2-3 months, and they last about 1-2 minutes before they resolve. She is also very warm when sleeping at night. She denies weight changes, palpitations, headaches, galactorrhea, acne, bowel changes, hair loss, and any changes to her skin or nails. Overall, she reports some mild malaise and irritability, but she denies depression and fatigue. Her past medical history is unremarkable, with no known medical conditions, no allergies, no medications, and no prior surgeries. She denies the use of alcohol, tobacco, and recreational drugs. Her method of contraception is vasectomy in her husband. Her last menstrual period was about 4 months ago, but periods were regular and monthly prior to that. Physical exam and vitals are normal. Test results are shown. Complete blood count (CBC)NormalThyroid stimulating hormone (TSH)NormalUrinalysisNormalComplete Metabolic Panel (CMP)NormalFollicle stimulating hormoneElevatedQuantitative hCGNegativeProlactinNormalFree/total testosteroneNormal Question You plan to educate the patient about her condition and offer treatment for her current symptoms. The patient should be counseled that her current diagnosis puts her at increased for what condition? Answer Choices 1 Alzheimer's disease 2 Endometrial hyperplasia 3 Endometriosis 4 Metastasis 5 Osteoporosis

Osteoporosis Explanation This patient is experiencing the climacteric stage of life, often referred to as menopause. Technically, menopause is defined as when a woman has had no menstrual periods for 1 year due to decreased ovarian function. The average age for this is 51.5. This woman would be considered peri-menopausal. The most common presenting symptoms of this normal transition of the reproductive phase of life are hot flushes (flashes) and night sweats. The rise in the "upstream" follicle-stimulating hormone (from the pituitary) in response to very low ovarian output is easily measured. In most cases, with classic symptoms and a patient of age in her 40s to 50s, testing for menopause is not necessary. If the patient desires, or if additional symptoms, history or physical exam suggest another disorder, testing can be done. With the drop in natural estrogen production, menopausal woman can experience rapid bone loss and are at risk for osteoporosis. Many approaches can be used to address the risk, including prescription hormone therapy and/or bisphosphonates (if appropriate), weight-bearing activity, adequate calcium and vitamin D, and avoidance of tobacco products. Much has been debated about the role of menopause and prescription hormone therapy in the development of Alzheimer's disease. Currently, no definitive conclusions can be drawn that menopause is a definite risk factor for Alzheimer's. Endometrial hyperplasia is a condition in which the lining of the uterus (endometrium) over-proliferates, which can lead to malignancy. It is not associated with normal menopause. In fact, with the decline in natural estrogen production, the endometrium atrophies and becomes very thin. Endometriosis is another condition in which estrogen effects can worsen the disease process, and a drop in estrogen (as with menopause) typically improves symptoms. Risk of endometriosis drops with menopause. This patient does not have a malignant condition associated with metastasis. If the patient is placed on hormone therapy, she should be counseled about the risks of breast cancer, but her climacteric state does not incur a specific risk for metastasis.

Case A 29-year-old woman, G4P2011, LMP 9 months prior, presents with sudden onset of severe lower abdominal pain. The pain is sharp and tearing and was not preceded by the contractions she recalls from previous deliveries. She also endorses vaginal spotting prior to presentation. There is no history of prior medical problems or surgery, and she is on no medications. All previous deliveries were vaginal. She has smoked 1 pack of cigarettes a day over the past 10 years. She denies alcohol use but does admit to a remote history of heroin abuse by insufflation. On physical examination: Temp 99.4°F; BP 110/70 mm Hg; Pulse 85/min; RR 20/min. Pertinent findings on the PE were relegated to the pelvic exam; fundal height measures 39 cm, and there is profuse bleeding from the vagina. Fetal monitor shows contractions every minute with elevated baseline uterine tone. Fetal tachycardia is evident at 180 beats/minute, and late decelerations are also present. Question What is the most likely diagnosis? 1 Cervical cancer 2 Chorioamnionitis 3 Placenta previa 4 Placental abruption 5 Uterine rupture

Placental abruption Explanation This patient has placental abruption. Abruption is a leading cause of second- and third-trimester bleeding, and it causes significant maternal and neonatal morbidity and mortality. A meta-analysis has demonstrated that smoking increases the risk of abruption by 90%. Maternal and paternal smoking increases the risk of abruption twofold. Risk increases fivefold when both parents smoke. Women with a history of abruption have a 15% increased risk during future pregnancies. Placenta previa, cocaine use, preeclampsia, and preterm premature rupture of membranes (PPROM) are also associated with placental abruption. Placental abruption is defined as a complete or partial separation of the placenta prior to delivery. The incidence of abruption is 5-6 out of 1000 deliveries. Obvious vaginal bleeding occurs if the hemorrhage develops between the membranes and the uterus. Concealed presentations occur when blood collects behind the placenta. Classically, placental abruption presents as painful third-trimester bleeding. Emergent cesarean is generally indicated when the patient is not in labor and hemodynamic compromise is present. If delivery is imminent and the abruption is mild, vaginal delivery may be attempted. Although cervical cancer can coexist with pregnancy, it is very unlikely in this case. The severity of the signs and symptoms in this case would be incongruent with achieving 9 months gestation. Clinical features of chorioamnionitis include maternal fever and uterine tenderness in the presence of confirmed premature rupture of membrane (PROM). Profuse vaginal painful bleeding is not a finding. Placenta previa classically presents with painless third-trimester bleeding. Uterine rupture is theoretically possible, but it is not the most likely diagnosis. A uterine rupture typically occurs during active labor, most commonly in women who have had a previous cesarean section. As this patient is hemodynamically stable, had a sudden onset of pain (not preceded by contractions/early signs of labor), and has not had any previous c-section deliveries, this diagnosis is very unlikely.

A 23-year-old primigravida is referred to her obstetrician by a community nurse. The nurse noted two blood pressure readings of 150/90 and 154/90 taken 15 minutes apart. Physical examination reveals a uterus at roughly 24 weeks gestation and grade 2 pitting edema. Urine analysis is 1+ positive for albumin. A sonogram at 8 weeks gestation showed a single live intra-uterine gestation. She has had regular antenatal checkups and has no past history of hypertension or diabetes. Her parents and grandparents are hypertensive. She is unaware of whether her mother or grandmother faced similar problems during their pregnancies. Question What is the most likely initiating event for her condition? 1 Overactivation of renin-angiotensin system 2 Overproduction of B-HCG 3 Placental ischemia 4 Renal artery stenosis 5 Essential hypertension

Placental ischemia Explanation Placental ischemia is the initiating event. This patient has classic features of pregnancy-induced hypertension, as evidenced by the blood pressure readings, edema, time of onset beyond 20 weeks, and proteinuria. The initiating event appears to be abnormal cytotrophoblast invasion of the spiral arterioles, which leads to activation of the maternal vascular endothelium. The cascade that follows includes enhanced formation of endothelin and thromboxane, increased vascular sensitivity to angiotensin II, reduced nitric oxide, and prostacyclin (vasodilator) synthesis. Overactivation of her renin-angiotensin system is an important component of the response, but it does not initiate the process. Overproduction of B-HCG may result in high blood pressure, as seen in hydatiform mole and multiple gestations, but there is no indication of either in this patient. Renal artery stenosis is a common cause of hypertension in young individuals. It would be present through the duration of pregnancy. Essential hypertension is a pathology associated with aging, the pathogenesis of which is unclear. It is unlikely in a 23-year-old, and the patient would have a past history of hypertension.

Case A 30-year-old woman presents with a 2-week history of vaginal discharge. She denies vaginal burning or itching, urinary frequency, hesitancy, and dysuria. The discharge is reported as light yellow in color. She is hesitant to provide much detail, so a physical exam is performed. On physical exam, the patient is in no distress, is afebrile, has normal vitals, and the exam is normal except for the pelvis. On speculum examination, the cervix is inflamed, bright red, and slightly friable with yellow discharge at the os. Vaginal mucosa is pink, moist, and without inflammation. Uterus and ovaries palpate to normal size and are non-tender on bimanual exam. Samples are collected for analysis and the results that are available are listed. TestResultUrine hCGNegativeUrinalysis (dipstick)Normal, except for 1+ WBCWet mount/KOH slideNegative for trichomonads, hyphae, and clue cells, >10 WBC per high-powered fieldComplete blood countNormal Question What is the most appropriate intervention for this patient given the most likely diagnosis? Answer Choices 1 Initiate twice weekly vaginal estrogen 2 Prescribe an antifungal (e.g., fluconazole) 3 Prescribe empiric antibiotics 4 Prescribe oral or intravaginal metronidazole 5 Recommend douching with vinegar solution

Prescribe empiric antibiotics Explanation This patient presents with cervicitis, an inflammation of the cervix characterized by mucopurulent cervical discharge or endocervical bleeding (induced by swabbing the os). Cervicitis, especially with the finding of increased white blood cells (WBCs) in the vaginal fluid, is associated with sexually transmitted diseases (e.g., Chlamydia trachomatis and Neisseria gonorrhoeae). The increased vaginal leucorrhea may also explain the (mildly) positive leukocyte esterase on the urinalysis dipstick. Cervicitis may occur on its own or as part of pelvic inflammatory disease (PID). Of the choices listed, the most appropriate intervention is to prescribe empiric antibiotics to cover Chlamydia trachomatis and Neisseria gonorrhoeae. Test results for these two infections are not readily available at the bedside, but treatment is recommended when the clinical diagnosis is made rather than delaying and possibly missing an opportunity to treat and prevent sequela. If the patient were menopausal, with vulvovaginal atrophy as the cause of her symptoms, the most appropriate treatment would be to initiate twice weekly vaginal estrogen. Common symptoms of atrophy include dryness, irritation, and dyspareunia. On physical exam, both the vulva and vagina are noted to be pale and dry. The tissue may also be friable, but the cervix would not be inflamed. If this patient's signs and symptoms were caused by a vaginal yeast infection (candidiasis), it would be reasonable to prescribe an antifungal (e.g., fluconazole). Common yeast infection symptoms include vaginal burning and pruritus; the exam reveals inflamed vaginal mucosa and thick white discharge. The wet mount slide with KOH (potassium hydroxide) would reveal the characteristic branching hyphae. If this patient's symptoms were caused by bacterial vaginosis (BV), it would be best to prescribe oral or intravaginal metronidazole. BV is characterized by a thin and watery vaginal discharge and fishy odor. Clue cells are seen on the wet mount of vaginal secretions. There are currently no gynecologic conditions for which experts recommend douching with a vinegar solution. Douches used to be quite popular and are still readily available over-the-counter, but they have little utility.

Case A 16-year-old girl has had recent onset of painful menstrual periods. Menarche was at age 12. Her periods were irregular for about 8-10 months. They have been fairly regular since then, with occasional mild crampy pains on the first 2 days. She rates the pains now as being 8-9 on a scale of 10. Cramps will start about a half a day before the onset of her periods, worsen the following day, and then gradually subside over the next day. She also describes having a headache, looser stools, mild nausea, and low back and thigh pain. Bleeding is moderate on the first 2 days. She is otherwise healthy without history of abdominal surgery. Privately she denies any sexual activity. She is an average student and has missed 1-2 days of school with each period for the past 3 months. She also denies any fever or dysuria. There is also no family history of gynecological problems. On exam, she has diffuse midline lower abdominal tenderness with some mild soreness to lower back and thighs, bowel sounds are normal, and there is no rebound tenderness. An external genital exam is normal. Question What is the most likely diagnosis? Answer Choices 1 Primary dysmenorrhea 2 Endometriosis 3 Irritable bowel syndrome 4 Imperforate hymen 5 Ovarian cysts

Primary dysmenorrhea Explanation Primary dysmenorrhea is associated with ovulatory cycles and typically occurs at an earlier age than secondary dysmenorrhea. Disability is common, with a reported 14% of girls reporting missing school or work because of pain, and nearly 50% of those have pain described as moderate or severe. Defining symptoms include crampy midline lower abdominal pain beginning with menstrual flow or shortly before. Cramps are typically intense on the first or second day and resolve before the end of the menstrual flow. The pain may be referred to the lower back and anterior thigh. Nausea or vomiting may occur. Management involves the use of NSAIDs, which reduce the production of prostaglandins. Endometriosis is a type of secondary dysmenorrhea. Functioning ectopic endometrial glands and stroma are present outside the uterine cavity, causing pain, irregular bleeding, and infertility. It occurs more frequently in first-degree relatives of women whose endometriosis has been confirmed surgically. The pain of irritable bowel syndrome in adolescents is similar to that described in adults, with paroxysmal periumbilical abdominal pain relieved by defecation or associated with an irregular pattern of defecation, including a change in frequency or consistency of stool, urgency, feeling of incomplete evacuation, and abdominal distention. Imperforate hymen is recognized as a membrane covering the vaginal opening seen on external exam. This may be bulging or a have a bluish hue from blood retained. Ovarian cysts are a type of secondary dysmenorrhea. Cysts are fluid- or semisolid-filled sacs that develop on or within the ovary. This may cause a sudden onset of pelvic pain to occur. There may be abnormal uterine bleeding, abdominal bloating or distention, or no symptoms.

Case A 14-year-old boy presents with bilaterally tender and swollen breasts. He has become increasingly self-conscious about the condition, resulting in his avoiding physical education classes and swim team practices. He has always had good general health with no history of medical illness, hospitalizations, or medications. He does not smoke or drink alcohol, and there is no history of illicit drug use or use of any activity-enhancing products. Family history is positive for a sister and his mother being diagnosed with breast cancer. The remainder of the review of systems is non-contributory. On physical examination, he appears normal in growth and development; there are tender soft masses in the lower quadrants of both breasts, approximately 3 centimeters in diameter. The remainder of the physical examination is unremarkable. Question What is the most appropriate next step in managing this patient? Answer Choices 1 Needle aspiration to rule out tumor 2 Reassurance that this is normal 3 Referral to a breast surgeon for biopsy 4 Referral to social services 5 2-week course of oral steroids

Reassurance that this is normal Explanation Based on the history and physical examination, the most likely diagnosis is pubertal gynecomastia, so reassurance is the best answer. Breast development in boys commonly happens during adolescence and may have many possible causes. Gynecomastia is a common condition that affects 40-60% of all boys/men. This breast growth can have a major emotional impact on young boys. The typical behavior is for them to hide the chest when in public due to embarrassment and/or fear of being ridiculed. During growth, this mass is most frequently tender and usually bilateral. Male gynecomastia during puberty is usually secondary to the normal hormonal imbalances between testosterone and estrogen that commonly occur during puberty, hence the name pubertal gynecomastia. In these cases, the breast tissue growth is usually less than 4 centimeters in diameter and will disappear without any therapeutic intervention within 2 years in 75% of these boys and within 3 years in 90%. Although usually normal, boys with gynecomastia should still have an evaluation by a physician, particularly if they are experiencing undue emotional distress because of it. The physician should reassure the teen that this is normal and should disappear within a few months or years without treatment in most cases. If gynecomastia is found in a boy under 9 who has not yet started puberty, more ominous causes must be considered. Medications, drugs, and diseases can all cause gynecomastia in boys/men and must be considered in your differential diagnosis. The most common medications are estrogen and anabolic steroids. Illicit drugs, such as marijuana, heroin, and alcohol, can also cause gynecomastia. Diseases that can cause gynecomastia include endocrine disorders, tumors, and genetic conditions such as Klinefelter's syndrome. In these cases, early recognition and appropriate treatment are most important. This is not a typical finding that would be consistent with non-accidental injury; a referral to social services would be inappropriate. Referral to a breast surgeon is unjustified in this case. Needle aspiration is unjustified in this case and may even be deleterious to growing breast tissue. Short course steroid therapy is unwarranted and not indicated in cases of pubertal gynecomastia.

Case A 74-year-old multiparous woman with a history of breast cancer has been referred to a gynecologist due to a sensation of vaginal fullness and pressure, sacral back pain with standing, coital difficulty, lower abdominal discomfort, and urinary frequency and incontinence. She states that she feels "a bulge" in the lower frontal vaginal area. She denies any fever, chills, flank pain, history of infectious diseases, changes in weight, dysuria, or hematuria. The physical exam is remarkable for an obese body habitus and non-tender bulge located anterior within the vaginal introitus that becomes more pronounced upon Valsalva and standing. Question What is the most appropriate next step in the management of this patient? 1 Advise maintenance of current body weight. 2 Apply topical estrogen cream. 3 Perform anterior colporrhaphy. 4 Recommend pelvic muscle exercises. 5 Refer for dynamic MRI defecography.

Recommend pelvic muscle exercises. Explanation This patient's most likely diagnosis is anterior vaginal wall prolapse or cystocele. Non-surgical (conservative) management of pelvic organ prolapse should be attempted before surgery is contemplated, as recommended by the Agency for Health Care Policy and Research and the ACOG Committee on Practice Bulletin. Conservative management is safe and inexpensive not usually associated with morbidity and mortality minimally invasive likely to lead to high patient satisfaction an option for patients awaiting surgery or patients who are not interested in surgical management Pelvic muscle exercises are an example of a conservative measure; they can improve pelvic floor muscle tone and stress urinary incontinence. The purpose of anterior colporrhaphy (anterior vaginal repair) is to plicate the vaginal muscularis fascia overlying the bladder (pubocervical fascia), diminishing bladder and anterior vaginal protrusion. This is indicated for patients with a central vaginal defect especially. Vaginal support devices (pessaries) are manufactured from medical-grade silicone; they are safe, cost-effective, and minimally invasive in treating patients with pelvic organ prolapse. Applying topical estrogen prior to pessary use is an important adjunct, particularly with signs of hypoestrogenism (atrophic vaginitis). Once the pessary is in place, as long as estrogen is not contraindicated (such as in estrogen-dependent breast tumors), vaginal estrogen cream should be applied 1-2 times per week or vaginal estrogen ring should be applied once every 3 months. General measures also include weight loss in obese patients. Defecating proctography and proctosigmoidoscopy may benefit patients with defecatory dysfunction. Patients with motility disorders should undergo colonic transit studies. Dynamic MRI defecography can provide defecatory and high-quality soft tissue imaging, but the test is expensive and has not been shown to change clinical decision making.

Case A 16-year-old girl presents with concerns over her "growth." She feels ashamed and left out because she is the only girl in her class who has not reached menarche. Her mother reached menarche at age 11, and her younger sister at 10. She is sexually active with her boyfriend of 1 year; she has always used contraception. On examination, her vital signs are normal. Physical examination reveals rudimentary breast buds at Tanner stage 2 and an absence of pubic and axillary hair. A pelvic sonogram shows a normal-appearing vagina and uterus. An MRI of the brain shows a normal-appearing pituitary gland and hypothalamus, with agenesis of the olfactory bulb. Question What is the pathophysiology of this condition?. 1 End-organ resistance to insulin 2 Adrenal hypofunction with reduced adrenal sex steroid secretion 3 Chromosomal anomaly 4 Reduced gonadotropin-releasing hormone from hypothalamus 5 Atresia of part of genital outflow tract

Reduced gonadotropin-releasing hormone from hypothalamus Explanation Kallmann syndrome is a form of idiopathic hypogonadotropic hypogonadism resulting from decreased production or action of gonadotropin-releasing hormone (GnRH) from the hypothalamus. This results in decreased pituitary secretion of gonadotropins FSH and LH. Patients present with hypogonadism, infertility, and varying degrees of pubertal development. Multiple genetic mutations have been associated with Kallmann syndrome, most commonly the PAX1 gene. Boys/men are more commonly affected. The hallmark of the disorder is anosmia or severe hyposmia present from birth. MRI of the brain may reveal multiple different malformations of the olfactory bulbs or sulci, including agenesis or partial agenesis. The pituitary and hypothalamus are structurally normal. It was previously thought to be associated with color blindness, now disproven. End-organ resistance to insulin is seen in polycystic ovarian syndrome. Pathogenesis is multifactorial: increased LH/FSH ratio, increased androgens, hirsutism, and polycystic ovaries. Menarche is not usually delayed, and sonogram studies show a "chain of lakes" picture of multiple ovarian cysts. Adrenal sex steroids are not primarily responsible for puberty. Chromosomal anomalies resulting in delayed puberty are most commonly seen in Turner syndrome. The XO genotype results in primary amenorrhea, failure of secondary sex characteristics, and streak ovaries. Coarctation of the aorta and renal abnormalities may be seen. Olfactory problems are not seen. Mullerian agenesis results in varying degrees of atresia of the outflow tract. It results from defects in development in fusion and development of the Mullerian ducts with the fallopian tubes, uterus, and the upper third of the vagina. The pelvic sonogram shows a normal uterus and tubes.

Case A 29-year-old man and his wife have been trying to have a child for the past 3 years. The wife was thoroughly evaluated for infertility; the workup revealed no abnormalities. Examination of the man's inguinal region reveals an ill-defined tortuous swelling that increases with standing and coughing. Ultrasonography is suggestive of varicocele. His initial semen analysis reveals mild oligospermia. Question What is the next step in management? Answer Choices 1 Embolization of dilated veins 2 Surgical mesh repair 3 Low-dose testosterone 4 Repeat semen analysis 5 Surgical excision of dilated veins

Repeat semen analysis Explanation The results of a single semen analysis are inadequate in making a diagnosis of infertility. For a multitude of reasons, counts vary over time. 2 or 3 separate counts should be taken at least 2-4 weeks apart. Despite the presence of a varicocele, if a repeat semen analysis is normal and the varicocele remains asymptomatic, there is no reason for operative intervention. Embolization of dilated veins is an option in the treatment of varicocele. The dilated veins of the pampiniform plexus are embolized using interventional radiology. It is used in the presence of a symptomatic varicocele or if infertility is confirmed. Surgical mesh repair is used in the management of a hernia, not in a varicocele. Low dose testosterone may be an option if infertility is due to hypogonadism due to low testosterone. It is not an initial step in management. If the varicocele is symptomatic or if infertility is confirmed, surgical excision is an option.

Case A 44-year-old woman G5P5 presents for her annual pelvic examination. Her menses are regular, and she is currently mid-cycle. She notes "leaking urine" when she coughs, sneezes, or strains. There is a bulge into the anterior vaginal wall; it is exacerbated when the patient is asked to "bear down." A urinalysis is unremarkable. There is no cervical motion tenderness or discharge noted on pelvic examination. The patient is afebrile and in no distress. No lesions are noted on the external genitalia and the pelvic examination is unremarkable except for the noted bulge. Vaginal cultures for gonococcus (GC) and chlamydia are pending. Question What is the most likely diagnosis? Answer Choices 1 Bladder infection 2 Menopause 3 Multiple sclerosis 4 Pelvic inflammatory disease 5 Stress urinary incontinence

Stress urinary incontinence Explanation The patient most likely has stress urinary incontinence as the result of a cystocele. A cystocele is a weakening and stretching of the wall between the bladder and vagina. This condition may cause leakage of urine (especially with coughing, sneezing, or exercising) or incomplete emptying of the bladder. Other names for this condition are "prolapsed bladder" or "fallen bladder." A cystocele is classified as grade 1 when the bladder is prolapsed partway into the vagina. It is grade 2 when the bladder reaches the opening of the vagina. It is grade 3 when the bladder bulges out through the vagina. A cystocele may be caused by childbirth or by excessive straining or lifting. A decreased amount of estrogen occurring during menopause may contribute to a weakening of the muscles that support the bladder and vagina. Treatment options may include avoidance of activities including heavy lifting, Kegel exercises, weight loss, a pessary (a ring-shaped device placed in the vagina that holds the bladder in place), or surgery to create support for the vagina by tightening the muscles and ligaments and repositioning the bladder. A bladder infection could cause symptoms of incontinence. This patient's urinalysis does not suggest infection, and she has not mentioned dysuria. This patient reports regular menses, so she does not meet the criteria for menopause. Menopausal women may have difficulty with incontinence as estrogen levels decrease. Estrogen helps to maintain the strength of muscles that control the bladder and keeps the lining of the vagina and urinary tract healthy. Incontinence and incomplete emptying of the bladder are both common symptoms in patients with multiple sclerosis because the disease involves a dysfunction of muscle coordination and strength. The patient does not give a history of previous symptoms suggestive of MS, but a workup may be warranted if symptoms worsen or persist without another obvious etiology. The presence of an anatomical defect suggests incontinence resulting from a cystocele in this patient. Pelvic inflammatory disease may also cause dysuria, often in association with cervical wall motion tenderness. GC and chlamydia cultures are pending; if there is a suspicion of PID, empiric treatment should be considered. There are no lesions consistent with herpes or chancroid, and the pelvic examination does not suggest the presence of an infection.

Case A 30-year-old woman presents with a 2-week history of vaginal discharge. She denies vaginal burning or itching, urinary frequency, hesitancy, and dysuria. The discharge is reported as light yellow in color. She is hesitant to provide much detail, so a physical exam is performed. On physical exam, the patient is in no distress, is afebrile, has normal vitals, and the exam is normal except for the pelvis. On speculum examination, the cervix is inflamed, bright red, and slightly friable with yellow discharge at the os. Vaginal mucosa is pink, moist, and without inflammation. Uterus and ovaries palpate to normal size and are non-tender on bimanual exam. Samples are collected for analysis and the results that are available are listed. Question If the patient provided full details of her history, what would be most consistent with this her presentation and likely condition? 1 Delayed menarche 2 Diabetes and virginity 3 Menopause 4 Recent antibiotic use 5 Unprotected vaginal intercourse

Unprotected vaginal intercourse Explanation This patient presents with cervicitis, an inflammation of the cervix characterized by mucopurulent cervical discharge or endocervical bleeding (induced by swabbing the os). Cervicitis, especially with the finding of increased white blood cells (WBCs) in the vaginal fluid, is associated with sexually transmitted diseases (e.g., Chlamydia trachomatis and Neisseria gonorrhoeae). The increased vaginal leucorrhea may also explain the (mildly) positive leukocyte esterase on the urinalysis dipstick. Cervicitis may occur on its own or as part of pelvic inflammatory disease (PID). Of the choices listed, the most likely history of this patient would be unprotected vaginal intercourse. It is common to obtain somewhat incomplete sexual histories due to patient embarrassment, distrust, or other factors. Delayed menarche is not associated with cervicitis. The approximate average age for menarche is 12.4. Genetic, anatomic, and hypothalamic/pituitary causes should be evaluated for girls over this age with primary amenorrhea. In a patient with diabetes who is a virgin and reports vaginal discharge, the most common consideration is a vaginal yeast infection (candidiasis). Common symptoms include vaginal burning and pruritus; the exam reveals inflamed vaginal mucosa and thick white discharge. The wet mount slide with KOH (potassium hydroxide) reveals the characteristic branching hyphae. In a menopausal woman, common vaginal symptoms include dryness, irritation, and dyspareunia. On physical exam, both the vulva and vagina are noted to be pale and dry. The tissue may also be friable, but the cervix would not be inflamed. A history of recent antibiotic use and vaginal discharge would suggest a possible vaginal yeast infection.

Case A 28-year-old primipara woman presents with wide fever swings 6 days after the delivery of a healthy male infant. Her pregnancy was uneventful, but because of the prolonged membrane rupture and prolonged labor, an emergency cesarean section was performed. On the second postpartum day, she started having fever and reporting uterine tenderness. She was treated with IV gentamycin and clindamycin. Today, her physical examination is normal; there is no uterine tenderness, and her WBC count is 11500, but she continues to spike fevers up to 39°C. Question What is the most likely diagnosis at this point? 1 Endometritis 2 Septic shock 3 Thrombophlebitis 4 Appendicitis 5 Drug fever

Thrombophlebitis Explanation This patient most probably has septic thrombophlebitis. It appears in day 5-6 in patients at risk (like those who had an emergency cesarean section after prolonged membrane rupture and prolonged labor). The patient has prolonged fever swings despite broad-spectrum antibiotics, and she has a normal physical and pelvic examination. Endometritis usually appears 2-3 days postpartum in patients with similar risk factors to those of thrombophlebitis, but patients present with clinical signs of exquisite uterine tenderness. Therapy that covers polymicrobial flora should result in the improvement of all signs and symptoms, including fever. This patient developed symptoms 6 days after the surgery, and all signs disappeared except a fever. She initially had endometritis, a risk factor for developing septic thrombophlebitis. Septic shock will have a dramatic clinical picture of acute circulatory failure that is unexplained by other causes. Symptoms and signs of septic shock are not specific (fever, chills, rigors, altered mental status, dyspnea, fatigue, malaise, nausea, vomiting, hypotension, and/or signs of tissue hypoperfusion), but this patient does not have symptoms or signs of circulatory failure. Appendicitis usually presents with RLQ abdominal pain, loss of appetite, nausea, vomiting, constipation or diarrhea, an inability to pass gas, low-grade fever, and abdominal swelling. This patient does not have abdominal tenderness or high fever. Drug fever is a febrile response that coincides temporally with the administration of a drug, most often 7-10 days after starting the drug, and it disappears after its discontinuation. To make a diagnosis of drug fever, you have to exclude other causes, so you should obtain the image of pelvic structures before making a diagnosis.

Case A 28-year-old woman at 24 weeks gestation presents with insomnia and fatigue. Her symptoms have worsened as the pregnancy has progressed. She has noticed dyspnea on exertion that is associated with palpitations when grocery shopping or going to the laundromat. Vital signs are blood pressure of 128/88 mm Hg, pulse is 102/minute, respirations are 16/minute, and temperature is afebrile. Weight is 138 lb, which is stable from her last visit 4 weeks ago. On physical exam, the patient is a gravid woman in no acute distress. Clinical findings include mild periorbital edema without ankle edema and brisk ankle reflexes. Examination is otherwise unremarkable. Question What will confirm the most likely diagnosis? 1 Cardiac stress test 2 Psychiatric evaluation 3 Thyroid function studies 4 CBC with iron studies 5 Urinalysis

Thyroid function studies Explanation Thyroid function studies, including a TSH and Free T4, confirm gestational thyrotoxicosis, commonly due to Graves' disease. Pregnancy-related fatigue during the first trimester typically improves by the second trimester. Patients with thyrotoxicosis present similarly to this patient and with insomnia secondary to hyperactivity, palpitations, and easy fatigue with exertion. Typically, thyrotoxicosis presents with weight loss, but a weight gain <0.9 kg/month is inadequate at this point in the pregnancy. Physical exam often reveals sinus tachycardia, periorbital edema in Graves' ophthalmopathy with or without proptosis, and hyperreflexia. Urinalysis would confirm the presence of proteinuria as seen in preeclampsia—diagnosed when a gravid woman presents with new-onset hypertension (140/90 mm Hg) in the presence of proteinuria. This diagnosis would be less likely than thyrotoxicosis because the patient's blood pressure is not this high, and it would not explain the patient's sinus tachycardia. A CBC with iron studies would identify an iron deficiency anemia, which could present with fatigue, dyspnea, tachycardia, and lack of weight gain if she had poor oral intake; it would not explain insomnia or hyperreflexia. Palpitations, shortness of breath, rapid heart rate, and shakiness could be seen if the patient was experiencing panic attacks. A psychiatric evaluation could establish this diagnosis, but it would not explain the patient's resting tachycardia, lack of weight gain in pregnancy, periorbital edema, or hyperreflexia. A cardiac stress test would identify a cardiac abnormality, such as coronary artery disease or arrhythmia. These would not explain the patient's periorbital edema, resting tachycardia, lack of weight gain in pregnancy, or hyperreflexia. Peripartum cardiomyopathy should be considered, but it is rare before 36 weeks. Patients present with exertional dyspnea, orthopnea, PND, peripheral edema, and cough. Physical exam shows evidence of JVD, peripheral edema, and displaced apical impulse. An echocardiogram confirms diagnosis. Physical exam is not consistent with peripartum cardiomyopathy, and it would not explain this patient's insomnia, lack of weight gain, hyperreflexia, and periorbital edema.

Case A 32-year-old woman is 2 hours status post cesarean delivery of a singleton gestation at 38 weeks. The course of her pregnancy was uncomplicated. She presented in early labor, which became prolonged despite oxytocin infusion. A cesarean section was performed when she failed to progress in labor and there were signs of fetal distress. While in the recovery area, she begins to complain of nausea and lightheadedness. On exam, her heart rate is 133 beats per minute and blood pressure is 76/42 mm Hg. Significant vaginal bleeding is noted, and abdominal palpation reveals a soft "boggy" uterus. Question What is the most likely diagnosis? Answer Choices 1 Uterine prolapse 2 Placenta accreta 3 Intra-amniotic infection 4 Uterine atony 5 Uterine rupture

Uterine atony Explanation The most likely cause of bleeding in this patient is uterine atony. Uterine atony is when the uterine myometrium fails to contract following delivery. Contractions of the uterine wall after delivery normally tamponade bleeding by constricting uterine arterioles. Absence of this response allows for continued bleeding, which usually becomes evident soon after delivery. Bimanual uterine palpation revealing a soft "boggy" uterus typically confirms the diagnosis. Risk factors include uterine distention caused by multiple gestations or polyhydramnios, prolonged oxytocin use prior to delivery, placenta previa, retained products of conception and intrauterine amniotic infection. Treatment involves uterine massage and the use of medications that promote uterine contractions, including oxytocin, ergot alkaloids, and prostaglandins. Treatment by manual or surgical extraction of retained products of conception is necessary, as these prevent adequate uterine contraction/tamponade. Uterine prolapse (also called uterine inversion) describes the collapse of the uterine fundus through the cervical os. The uterus may be seen protruding from the vagina and abdominal exam reveals absence of the uterine body. It is a rare but serious event requiring rapid correction to restore hemodynamic stability. Placenta accreta describes a condition in which the placenta implants within the uterine myometrium. This is a rare cause of postpartum hemorrhage and would have been recognized at the time of cesarean delivery. Intra-amniotic infection (formerly known as chorioamnionitis) may impair myometrial contractility, which can result in labor abnormalities, need for cesarean delivery, uterine atony, and postpartum hemorrhage. Uterine rupture is also a rare complication involving a complete tear of the uterine myometrium with extrusion of uterine content into the abdominal cavity. This potentially fatal complication most commonly occurs at the site of a uterine scar from prior cesarean section and requires an emergent laparotomy for delivery of the fetus and stabilization of the mother.

Case A 32-year-old woman with no significant past medical history presents with a 4-day history of vaginal discharge. She describes the vaginal discharge as thin, fairly uniform in its consistency, and of a light grayish color. Her last sexual intercourse encounter was 1 week ago; she admits that it was unprotected. She is not in a monogamous relationship. She denies any fever, chills, swollen glands, dysuria, hematuria, urinary frequency, dyspareunia, or back pain. She further denies any vulvar or vaginal pruritus. Physical exam is significant for a pungent ammonia-like scent with an associated thin gray-white vaginal exudate, but it is otherwise unremarkable. Microscopic evaluation of the vaginal exudates is remarkable for the presence of clue cells. Question What additional laboratory finding would be expected in this patient? Answer Choices 1 Candida spores and hyphae on vaginal wet mount 2 KOH whiff test negative 3 Vaginal pH of 5.2 4 Numerous white blood cells within the vaginal exudate 5 Red blood cell casts upon urinalysis

Vaginal pH of 5.2 Explanation This patient's signs and symptoms are characteristic of bacterial vaginosis. While the infection is polymicrobial in etiology, an overabundance of Gardnerella vaginalis and other anaerobes are causative. Increased malodorous discharge without pruritus, dysuria, or inflammatory changes is typical for the infection. White blood cells are not numerous when vaginal exudate is examined microscopically. This is not a sexually transmitted infection. Clinical criteria for diagnoses include homogeneous white, noninflammatory discharge microscopic presence of > 20% clue cells vaginal discharge with pH greater than 4.5 a fishy odor with or without addition of 10% potassium hydroxide (KOH) Furthermore, Gram staining will reveal small, non-motile, non-encapsulated, and pleomorphic rods. Candida spores and hyphae on a vaginal wet mount would be consistent with a yeast infection, rather than with BV. Red blood cell casts are not typical of bacterial vaginitis; their presence should suggest acute glomerulonephritis, lupus nephritis, SBE, Goodpasture disease, aftermath of streptococcal infection (poststreptococcal glomerulonephritis), vasculitis, or malignant hypertension.

Case A 64-year-old woman presents with urine leakage that has been progressively worsening for the past 6-12 months. She also admits to having vaginal dryness, poor lubrication with intercourse, and subsequent dyspareunia. Urinary leakage primarily seems to occur with coughing, sneezing, or high-impact exercise. She denies dysuria, hematuria, urgency, and vasomotor symptoms. She tries to empty her bladder every 2-3 hours and has cut out all alcoholic and caffeinated beverages, but symptoms persist. The patient is a G6P6. She has no chronic medical conditions and no past surgeries. She wants to do everything she can to avoid surgery. She is in a monogamous relationship. She was last seen 1 year ago with a normal well-woman exam and normal Pap smear. On physical exam, vitals are normal. The pelvic exam reveals dry pale vaginal mucosa with a bulge on the anterior vaginal wall that increases in size with a Valsalva maneuver. Urinalysis is normal. Question Given this patient's strong preference to avoid surgery, what else could be done to improve her urinary symptoms? Answer Choices 1 Vaginal pessary 2 Oral nitrofurantoin (Macrobid) 3 Oral pentosan polysulfate sodium (Elmiron) 4 Oral phenazopyridine (Pyridium or Azo) 5 Oral tolterodine (Detrol)

Vaginal pessary Explanation This patient's urinary symptoms are secondary to a cystocele, which is a bulging of the bladder into the vagina. This patient's pelvic floor musculature has been weakened by delivery of 6 children, and her symptoms are further aggravated by postmenopausal vaginal atrophy. Stress incontinence is one of the most common symptoms associated with having a cystocele. If she is a good candidate for surgery, the most effective treatment for this patient's cystocele is surgical repair. Given her preference to avoid surgery, however, a pessary is the best non-surgical treatment and can decrease symptoms of prolapse (bulge, pressure, splinting) and associated urinary symptoms (stress and urgency incontinence and voiding difficulty). Topical estrogen cream may also help with the vaginal dryness and might provide some mild relief of urinary symptoms, but it does not address the root cause of the problem. Oral nitrofurantoin (Macrobid) is an antibiotic, which would be appropriate for treatment (or prophylaxis) of urinary tract infection (UTI). This patient does not present with typical UTI symptoms of dysuria, frequency, and urgency, and she does not have an abnormal urinalysis. Oral pentosan polysulfate sodium (Elmiron) is a medication used for interstitial cystitis (IC), a chronic inflammatory disorder of the bladder that classically presents with nocturia, urgency, frequency, pain with a full bladder, and sometimes dyspareunia. IC is a diagnosis of exclusion. This patient has a physical exam finding of a cystocele and vaginal atrophy, which explains her symptoms. Oral phenazopyridine (Pyridium or Azo) is a medication frequently used for relief of dysuria associated with UTI. This patient denies dysuria. Oral tolterodine (Detrol) is an anticholinergic medication used for urge incontinence and overactive bladder (OAB) syndrome. This patient denies urge incontinence and OAB symptoms.

A 56-year-old Asian woman presents to establish care for a wellness exam. She recently relocated from out of state; she brings a copy of her medical notes and labs from her previous medical provider. The patient tells you that sexual intercourse is very uncomfortable for her. She admits insertional dyspareunia and poor lubrication, but she denies deep dyspareunia. Outside of intercourse, she denies pelvic issues. She denies vaginal bleeding, pruritus, or discharge. She also denies urinary issues, such as dysuria, frequency, and incontinence. She has occasional hot flashes, but she feels they are tolerable. Her relationship is otherwise healthy, and she does not feel pressured into intercourse against her will. She is a G2P1. Surgical history includes open fracture reduction of the left ankle at age 22 and a dilatation and curettage (D&C) at age 24 for a spontaneous abortion (SAB). She is divorced, and she has been in her current monogamous relationship for 1.5 years. She is a non-drinker and non-smoker; she denies recreational drug use. Family history is unremarkable. The table indicates laboratory tests that were done approximately 1 year ago. Question Based on this patient's history and lab results, what physical exam finding do you expect? Answer Choices 1 Thick white discharge adhering to vaginal mucosa 2 Uterine enlargement 3 Uterine prolapse 4 Vulvar and vaginal atrophy 5 Yellow-green vaginal discharge

Vulvar and vaginal atrophyExplanation This patient would be expected to have vulvar and vaginal atrophy on her physical exam. Vulvar and vaginal atrophy occurs after menopause when the estrogen receptors in the epithelium are no longer stimulated as a result of declining ovarian output of sex steroids. With vulvar and vaginal atrophy, the clinician will observe thinning of the labia, less rugation of the vaginal walls, and pale coloration with dry (and possibly friable) vaginal mucosa. This condition occurs gradually after menopause, and patients with this condition may be reluctant to discuss it. Common complaints are vaginal dryness, irritation, dyspareunia, and inadequate lubrication with sexual activity. While routine testing for menopause is not recommended, this patient's laboratory results show an elevated follicle-stimulating hormone (FSH) and luteinizing hormone (LH), with low estradiol, which is consistent with her status as postmenopausal. A thick white discharge adhering to the vaginal mucosa would be the physical exam finding expected in a woman with a vaginal yeast infection (candidiasis). Patients might report vaginal irritation, pruritus, and increased discharge. She may also report insertional dyspareunia. This patient denied the other symptoms and is clearly postmenopausal, putting her at risk for atrophy. Uterine enlargement can be seen in women with uterine fibroids (leiomyomas) or simply after having multiple pregnancies. Uterine fibroids tend to diminish in size after menopause and this patient only experienced two pregnancies. She would not be expected to have appreciable uterine enlargement based on her history and labs. Uterine prolapse may occur in younger women, but it is most common in postmenopausal women. The pelvic floor support and uterine ligaments are inadequate to keep the uterus in place and it descends into the vagina. Prolapse can vary in severity. When it is present, the patient may describe a feeling of vaginal fullness or that "something is falling out." Prolapse is not associated with insertional dyspareunia or poor vaginal lubrication. Yellow-green vaginal discharge indicates vaginal infection. This patient has been tested for sexually transmitted diseases with her current partner, and she currently denies vaginal discharge and pruritus. With her condition of atrophy, the physical exam will reveal less-than-normal vaginal discharge.

Case A 62-year-old woman presents due to urine leakage for 2 years that has worsened. She leaks urine when she coughs or sneezes; she wears a pad daily as a result. The amount of leakage varies. The patient denies hematuria, dysuria, and pelvic pain. She sometimes feels vaginal pressure and fullness. She is considering quitting her job from embarrassment. Past medical history includes 4 vaginal deliveries. No other known medical conditions, medications, or allergies. She is married and works part-time at a call center; she denies tobacco, alcohol, and recreational drugs. On physical exam, she is obese, with an atrophic vulva/vagina. Pelvic examination reveals downward and forward rotation of the vaginal wall, with an anterior bulging when the patient is asked to strain. A dipstick urinalysis is normal. Question After completing treatment of her condition, what is the most appropriate way of preventing recurrence? 1 Avoidance of prolonged sitting 2 Daily cranberry supplements 3 Daily prophylactic antibiotics 4 Oral or transdermal oxybutynin 5 Weight loss

Weight loss Explanation This patient has a cystocele, a herniation of the bladder wall into the vagina. Common symptoms include stress urinary incontinence (SUI), a feeling of vaginal fullness, incomplete voiding, and dyspareunia. Treatment options include surgical repair, pelvic floor physical therapy, and use of a pessary. Several risk factors (obesity, chronic cough, multiple vaginal births) have been identified with cystoceles. In this obese patient, weight loss is recommended to prevent recurrence of cystocele after treatment. Frequent straining, which increases intra-abdominal pressures, may contribute to cystoceles. If this patient had a job with frequent heavy lifting, employment may need addressing. There is no reason to counsel her to avoid prolonged sitting at her job to prevent cystocele recurrence, but regular physical activity should be encouraged. Daily cranberry supplements are often used for urinary tract health. There is no evidence that cranberry juice or supplements are helpful for cystoceles and stress urinary incontinence. Daily prophylactic antibiotics can be helpful in managing patients prone to frequent pyelonephritis, cystitis, or urinary tract infection (UTI). Oral or transdermal oxybutynin, an anticholinergic, is used for detrusor instability (overactive bladder) and resulting urge urinary incontinence. No medications have been proven effective in preventing cystoceles.

Case A 54-year-old woman with diabetes mellitus presents with a 3-week history of vaginal spotting. An endometrial biopsy is performed and the pathology indicates endometrial cancer. The patient has a history of 12 years of amenorrhea and considers herself postmenopausal. She denies pelvic pain and cramping. She cannot identify any health changes that relate to the new vaginal bleeding. She has not seen a medical provider for preventive services for over 5 years due to a lack of health insurance. She is currently on metformin 500 mg TID; the patient admits poor compliance with the second and third doses each day. She has no known allergies, and her past medical history is significant for a cholecystectomy and tonsillectomy. The patient does not smoke or drink alcoholic beverages and denies illicit drug use. Menarche was at age 15; her menses were generally regular in her teens and 20s. She was amenorrheic while using depot medroxyprogesterone acetate, then menses returned but were irregular in frequency in her 30s. She believes her LMP to have been around age 42. She denies significant dysmenorrhea, menorrhagia, or premenstrual syndrome throughout her menstrual history. She is a G8P6Ab2, with her first child delivered at age 19; she had 6 spontaneous vaginal births with two first-trimester spontaneous abortions. She has had 9 sexual partners and has no history of sexually transmitted diseases. Previous methods of contraception included oral contraceptives (briefly) in her 20s and depot medroxyprogesterone acetate (DMPA) for 5 years in her early 30s. She discontinued the use of contraceptives in her mid-30s and has rarely been sexually active since. Question What intervention would have had the best effect in preventing this patient's development of endometrial cancer? Answer Choices 1 A progestin during times of amenorrhea if the patient was not already on contraceptives 2 Avoidance of depot medroxyprogesterone acetate (DMPA) injections for contraception 3 Avoidance of the oral contraceptives 4 Treatment of her diabetes with insulin instead of oral medication 5 Yearly Pap tests over the last 5 years

A progestin during times of amenorrhea if the patient was not already on contraceptives Explanation The correct response is administration of a progestin during times of amenorrhea if the patient was not already on contraceptives. This patient's long period of amenorrhea followed by postmenopausal bleeding is suggestive of progression from endometrial hyperplasia to endometrial cancer. Various types of hyperplasia (e.g., simple, complex, atypical simple, and atypical complex) can be treated with progestin, which inhibits and reverses endometrial hyperplasia.Depot medroxyprogesterone acetate is considered a type of progestin that will treat endometrial hyperplasia and prevent progression to cancer. Total avoidance of DMPA throughout the patient's life would not have protected her uterus.Oral contraceptives contain a progestin component that helps counter the endometrial overgrowth that leads to hyperplasia and cancer development. In fact, oral contraceptive use is considered a protective factor for the development of endometrial cancer. The avoidance of oral contraceptives in this patient may have hastened the development of her cancer.The patient's diabetes is a risk factor for the development of endometrial cancer, especially if combined with obesity, but the type of treatment for diabetes (oral medication or insulin injections) has not been clearly associated with better or worse outcomes in endometrial cancer. An improved degree of glucose control indicates a decreased risk of endometrial cancer compared to poor glucose control in a diabetic. Insulin tends to provide better glucose control if the patient is compliant with the treatment. In this patient who already admits poor compliance with oral diabetes medication (metformin), it is anticipated that her compliance would be similar or worse with insulin injections. No firm conclusion can be drawn that the type of diabetes treatment would have prevented endometrial cancer.Yearly Pap tests may or may not have detected abnormalities in pathology; the Pap is not considered a reliable screening test for detecting endometrial cancer. If atypical glandular cells of undetermined significance are detected on a Pap, then definitive evaluation (e.g., a dilatation and curettage or endometrial biopsy) should be pursued. Having regular Pap tests would not have prevented the development of endometrial cancer, but they potentially could have detected an abnormality at an earlier time.

Case A 52-year-old woman presents to her gynecologist's office with a 6-month history of hot flashes, night sweats, mood swings, and vaginal dryness that interferes with intercourse. The symptoms seem to be worsening and are now interfering with her productivity at work and with her relationships with family and friends. Her last normal menstrual period (LNMP) was 8 months ago. She denies tobacco use, and she drinks one glass of red wine daily. She has a history of coronary heart disease (CHD) with stent placement 2 years ago. She recently read an article about hormone replacement therapy (HRT). Question How should this patient be advised? 1 HRT is indicated for this patient's symptoms. 2 Estrogen replacement alone is indicated. 3 HRT will control hot flashes, but not vaginal dryness. 4 HRT is contraindicated due to regular alcohol use. 5 Antidepressant therapy is indicated for menopause-related mood swings.

Antidepressant therapy is indicated for menopause-related mood swings. Explanation Antidepressant therapy is indicated for this patient's menopause-related mood swings. This patient presents with classic symptoms of natural menopause with hot flashes, night sweats, mood swings, and vaginal dryness. Due to this patient's history of coronary artery disease and history of stenting, HRT is contraindicated. An alternative therapy option would be antidepressant therapy, such as an SSRI, for the treatment of this patient's mood swings. It would not result in relief of vasomotor symptoms or vaginal dryness. Estrogen replacement alone is contraindicated in this patient because her uterus is intact; estrogen unopposed by progesterone increases the risk of endometrial cancer. HRT will control all of her physical symptoms: vasomotor symptoms (hot flashes, night sweats) and vaginal dryness. Regular alcohol use is not a contraindication for hormone replacement in this patient, as her intake is moderate. Active liver disease is a contraindication to HRT.

Case A 26-year-old woman presents with dysmenorrhea that has become more bothersome over the last 3 months. She has difficulty taking ibuprofen (Motrin) due to a stomach ulcer. Question What is the most appropriate non-pharmacological measure that would relieve her symptoms? Answer Choices 1 Increasing caffeine intake 2 Limiting aerobic exercises 3 Applying warm compresses locally 4 Drinking a glass of wine at night 5 Limiting sexual activity

Applying warm compresses locally Explanation Warm compresses relieve cramping associated with dysmenorrhea by relaxing the spasm. In studies, heat is shown to be as effective as ibuprofen. Caffeine intake should be decreased to limit symptoms. Caffeine's diuretic properties contribute to cramping by increasing urination and dehydration. Engaging in aerobic exercise aids in mitigating symptoms by raising levels of endorphins, which may help reduce pain. Regular exercise is known to reduce stress, anxiety, depression and promote psychological well-being. Alcohol (wine) can also make symptoms worse, as it has similar effects to those of caffeine. Sexual activity and orgasm may relieve menses-related discomfort by having the same effects as exercise.

Case A 26-year-old woman at 29 weeks gestation presents with a 1-week history of vaginal discharge and dysuria. On genital exam, she has mucopurulent cervicitis. A cervical swab is obtained and results are positive for Chlamydia trachomatis infection. Question What is the best course of treatment? Answer Choices 1 Doxycycline 2 Metronidazole 3 Levofloxacin 4 Fluconazole 5 Azithromycin

Azithromycin Explanation Azithromycin is safe for use in pregnancy (Category B), and it is effective against chlamydia. Doxycycline is effective against chlamydia, but it is contraindicated for use in pregnancy (Category D). Levofloxacin is effective against chlamydia, but it is contraindicated in pregnancy (Category C). Metronidazole is safe for use in pregnancy, but it is not effective for treatment of chlamydia. Fluconazole is Category C for use in pregnancy and is not effective against chlamydia.

Case A 22-year-old woman presents with increased vaginal discharge. She is sexually active with two male partners and she uses birth control pills for contraception. Her last menstrual period was 12 days ago, and she has noticed an increased whitish vaginal discharge for the past week. Physical exam reveals a soft non-tender abdomen. On pelvic exam, she has a light-yellow cervical discharge with erythema of the cervical os. There is no cervical motion tenderness and no adnexal masses or tenderness. Wet mount of the vaginal discharge reveals epithelial cells and WBCs, and no yeast or protozoa. Gram stain of the vaginal discharge reveals many leukocytes. A urine pregnancy test is negative. Question What is the best treatment for this patient? 1 Azithromycin 1 g PO once and doxycycline 100 mg BID for 14 days 2 Ciprofloxin 500 mg PO once and ceftriaxone 125 mg IM once 3 Azithromycin 1 g PO once and Ciprofloxin 500 mg once 4 Azithromycin 1 g PO once and ceftriaxone 250 mg IM once 5 Ciprofloxin 500 mg PO once and metronidazole 2 g PO once

Azithromycin 1 g PO once and ceftriaxone 250 mg IM once Explanation This patient has a mucopurulent cervicitis on physical exam, which is usually caused by Neisseria gonorrhoeae or Chlamydia trachomatis. The patient should be treated with antimicrobial agents that will cover both pathogens because there is a high risk of co-infection with both pathogens (at least 50%). Patients should be tested for infection with both chlamydia and gonorrhea before treatment, usually with an immunoassay. In addition, a Gram stain of the cervical discharge can help elucidate the responsible pathogen. In 60% of women with gonorrhea, gram-negative intracellular diplococci may be identified. In chlamydial infections, the Gram stain will reveal many leukocytes (>10/high powered field) but no gonococci. These patients should also be screened for co-infection with syphilis and possibly HIV. This patient does not have signs of salpingitis or tubo-ovarian abscess (pelvic inflammatory disease). Specifically, she has no cervical motion tenderness or peritoneal signs and no adnexal tenderness or masses. She will qualify for outpatient treatment of cervicitis and can avoid the more aggressive treatment used for salpingitis. In either condition, both of the patient's sexual partners will require medical evaluation and treatment. Additionally, if either Neisseria gonorrhoeae or Chlamydia trachomatis is identified with culture or immunoassays, infection with these pathogens must be reported to the local health department. Acceptable regimens for the treatment of gonorrheal cervicitis include one-time treatment with ceftriaxone 250 mg IM once or cefixime 400 mg PO. Alternative regimens for gonorrheal cervicitis include ceftizoxime 500 mg IM; cefoxitin 2 g IM with oral probenecid 1 g; or cefotaxime 500 mg IM. Spectinomycin 2 mg IM is the recommended treatment for patients with penicillin or cephalosporin allergy, but spectinomycin is not currently available in the US. Previously, quinolones such as ciprofloxacin were an option for gonorrheal cervicitis, but these regimens are no longer recommended by the CDC due to increasing resistance to the quinolones in Asia and many metropolitan areas of the US. Additionally, all patients with cervicitis should receive a regimen to treat chlamydia. These regimens include azithromycin 1 g PO once or doxycycline 100 mg PO twice a day for 7 days. Metronidazole 2 g PO is the regimen used to treat vaginitis due to Trichomonas, not mucopurulent cervicitis.

Case A 45-year-old woman presents with a 3-day history of persistent nipple discharge. She is essentially healthy with an insignificant past medical history. She quit her job to stay home with her young children in the past year; she has noted a weight gain of 15 lb, which she attributes to being at home more, leading to increased snacking as well as less activity. She also says her periods have spaced out, occurring anytime from 28-53 days. They still last about 2-3 days, as they previously had. She denies hot flashes and the possibility of pregnancy; her husband had a vasectomy. Her review of systems is otherwise negative. She takes vitamin E every day, but she takes no other medications or herbs; she denies the use of illicit drugs. On physical exam, you note a healthy-looking woman. Her breast exam is negative for lumps, dimpling, and nipple retraction. You are able to express some clear fluid from the left nipple. It seems serous. Her axillary exam is negative for any enlarged lymph nodes; the rest of her exam is normal. Question What is the next step in the management of this patient? Answer Choices 1 Check an FSH. 2 Do microscopy of the fluid. 3 Check a prolactin level. 4 Order a mammogram. 5 Check a TSH.

Order a mammogram. Explanation A mammogram should be ordered. The patient is a 45-year-old woman with spontaneous serous discharge. Nipple discharge is a common breast complaint and is usually associated with benign etiology. Because most nipple discharge is a result of benign processes, less invasive, non-surgical diagnostic modalities have been explored to alleviate the need for surgical intervention. Most of the patients with nipple discharge have benign disease, such as intraductal papilloma, papillomatosis, duct ectasia, or fibrocystic condition. Studies have shown that approximately 11% of patients will have an underlying carcinoma. To be significant, nipple discharge should be spontaneous, persistent, and non-lactational. Persistent clear or bloody nipple discharge merits, at the very least, diagnostic breast imaging. All nipple discharges associated with a breast lump must be surgically evaluated. The incidence of associated cancer is much higher when the discharge is serosanguineous or sanguineous. Mammography has been utilized for evaluation of obvious abnormality to guide surgical intervention. The sensitivity of mammography for detecting malignant ductal pathology is 57% and the positive predictive value is 16.7%. It should be the initial test in patients with clinically significant nipple discharge and in women over 40. Cytological analysis of the discharge fluid has traditionally shown high false-negative rates, and it is not revealing in localizing the lesion. Hemoccult testing has been shown to be useful for determining the presence of blood, but its use in predicting histology is questionable. For the patient in this case, all the tests noted could be indicated. An FSH can be used to check for menopausal status, and a prolactin level could be done to check for abnormal levels. A TSH to check for hypothyroidism could also be done; decreased thyroid hormones are associated with galactorrhea. Hypothyroidism would also explain the menstrual irregularity. Mammography should be conducted initially; later, hormonal assay and a cytological examination can be carried out. In summary, given that this patient presented with spontaneous persistent discharge, the most appropriate initial approach would be to perform a mammography.

Case A 26-year-old woman presents to her gynecologist's office with a 4-month history of amenorrhea. She has had some breast tenderness, but she denies nausea, vomiting, fatigue, and abdominal pain. She was sexually active until about 6 weeks ago when she broke up with her boyfriend, but she states that they used condoms. Question What appearance of her cervix during an internal examination would support your suspicion? Answer Choices 1 Purulent discharge from cervical os 2 Strawberry-like appearance to cervix 3 Bluish appearance to cervix 4 Cervical os slightly open 5 Blood from cervical os

Bluish appearance to cervix Explanation This patient would be approximately 10 weeks pregnant and bluish appearance to the cervix is the correct answer. While some pregnant women can have a normal appearance to their cervix, most will have a bluish appearance. In addition, bimanual palpation will reveal the cervix to be soft and flexible. The uterus also increases in size with gestational age and will fill the pelvis around 12 weeks. Strawberry-like appearance to the cervix is more likely to be seen in patients who have some form of cervicitis. In particular, patients with cervicitis caused by trichomonas usually have purulent vaginal discharge and a friable cervix with punctate hemorrhages that give it a "strawberry-like" appearance. Cervical os slightly open is not the most likely appearance of the cervix but could be seen in the cervix of a pregnant female. The os is usually only slightly open if the patient is experiencing a spontaneous abortion, has an incompetent cervix, or is in the early stages of labor. Purulent discharge from the cervical os the likely signals an infection. Blood from cervical os is not typically indicative of a pregnant patient. Blood coming from the cervical os is more likely to be seen in a patient currently having their menses, a patient experiencing a spontaneous abortion, cervical malignancy, and during some stages of labor. If a female is known to be pregnant and has blood coming from the os, there are likely to be other symptoms associated with various disorders, such as placental abnormalities.

Case A 25-year-old woman presents with pelvic pain for the last 3-4 menstrual cycles, the most recent episode with increasing symptoms for approximately 2 weeks. She has tried over-the-counter pain relievers without relief. She admits deep dyspareunia, and she has been with her current sexual partner for about 6 months. She does not use barrier contraception every time with her current and previous partner, although she is on an oral contraceptive pill. She has no chronic medical conditions. There are no allergies to any medications. She denies urinary issues. She admits to nausea, but no vomiting or diarrhea. She reports a negative home pregnancy test this morning. Her last pelvic exam was over 1 year ago. LMP was 1 week ago. Question What physical exam findings would best narrow your diagnosis? Answer Choices 1 Abdominal guarding 2 Adnexa approximately 2.5 cm in size 3 Anteverted uterus 4 Cervical motion tenderness 5 Discomfort with speculum insertion

Cervical motion tenderness Explanation Cervical motion tenderness along with a history suspicious for pelvic inflammatory disease (PID) is a red flag for the diagnosis. While testing for sexually transmitted infections (especially Chlamydia trachomatis and Neisseria gonorrhea) is indicated, the diagnosis is typically based on history and physical exam. The Centers for Disease Control Sexually Transmitted Treatment Guidelines recommend empiric treatment for PID if one or more of the following minimum criteria are present on exam: cervical motion tenderness uterine tenderness adnexal tenderness A high index of suspicion is needed, and treatment should be initiated while awaiting laboratory results. The examiner should further elicit this patient's sexual partner history and potential vaginal/cervical discharge. Other conditions, such as endometritis and endometriosis, could cause cervical motion tenderness. The CDC guidelines are helpful in selecting antibiotic therapy and whether to treat as inpatient or outpatient. Most women will find that pelvic exam is uncomfortable, but tenderness with movement of the cervix is not a normal finding and indicates inflammation in the uterus. The remaining physical exam findings listed are less helpful than a finding of cervical motion tenderness. Abdominal guarding is a common physical exam finding with many gastrointestinal, urinary, and pelvic disorders. Guarding refers to voluntary or involuntary contraction of the abdominal musculature in response to pain felt during palpation. It is not a specific finding for pelvic inflammatory disease, and it would not help narrow a diagnosis as much as cervical motion tenderness. A physical exam finding of an adnexa approximately 2.5 cm in size is a normal physical exam finding. When the patient is thin enough for a good estimate of ovarian size, an examiner may notice enlargement in the case of ovarian cysts, ectopic pregnancy, tumors, and other ovarian disorders. A finding of normal ovarian size does not narrow the differential for this patient. An anteverted uterus on physical exam simply indicates the angle of the fundus points toward the ventral side of the patient. This is also a normal finding, and it does not assist in arriving at a definitive diagnosis. Discomfort with speculum insertion can be related to many factors, including examiner skill, patient anxiety, vaginal inflammation, and many gynecologic disorders. This finding is non-specific and also does not help with diagnosis.

Case A 22-year-old primiparous woman at 10 weeks gestation presents to the office. She works in an office, but she maintains a regimen of swimming and moderate weightlifting 3-4 times per week. She has had no vaginal spotting, and the pregnancy is progressing normally. She would like to continue with her exercise routine but is concerned about reports from friends that exercise has an adverse effect on pregnancy. Past medical history is otherwise unremarkable. Vital signs are normal, and examination is consistent with a 10-week gestation. Question How should this patient be advised? 1 Adopt a sedentary lifestyle during pregnancy. 2 Reduce vigorous activity to reduce miscarriage risk. 3 Decrease exercise to decrease prematurity risk. 4 Continue current exercise regimen. 5 Limit activity to normal activities of daily living.

Continue current exercise regimen. Explanation Exercise during pregnancy provides benefits to both mother and child. Pregnant women should engage in 2.5 hours of moderate-intensity exercise per week during pregnancy. Maternal benefits include increased pain tolerance, lower total weight gain, and a decreased risk of preeclampsia and gestational diabetes. Fetal benefits include decreases in the incidences of preterm birth, cesarean delivery, and large-for-gestational-age births. These recommendations apply to a normal pregnancy. Conversely, a sedentary lifestyle during pregnancy increases the risk of gestational diabetes, preeclampsia, peripartum complications, and post-pregnancy weight retention. Some activities should be avoided, such as those with a risk of abdominal trauma, scuba diving, or those in an overheated environment. Vigorous activity during pregnancy is not associated with an increased risk of miscarriage. Moderate-intensity and vigorous exercise actually decrease the rate of prematurity. Usual activities of daily living do not provide sufficient activity to achieve the benefits of 2.5 hours of moderate-intensity exercise per week during pregnancy.

A 31-year-old Caucasian woman presents for genital itching that she has been experiencing for a while. She has treated herself with OTC medication for a yeast infection, which has not helped. Now she notices a lump, which appears to be raised and nodular on exam. Her menstrual cycles have remained unchanged since puberty. She is G0P0 and has been taking oral contraceptives for the past 11 years. Her family and social history are insignificant. Question The presentation is most consistent with what type of cancer diagnosis? 1 Vulva 2 Vagina 3 Cervix 4 Endometrium 5 Ovary

Correct answer: Vulva Explanation The most common presenting symptom of vulvar cancer is pruritus (itching), which often results in a delay in diagnosis. Localized areas of color differences secondary to increased pigmentation, hyperkeratosis, wart-like lesions, and shallow ulcerations, are indicative of neoplastic changes of the vulva. Cancer of the vulva can occur at any age. More than 50% of the cases occur in women over 70. In recent years, the occurrence has been increasing in the 20-40 age group. 33% of affected women have associated papillomavirus infection of the cervix, vagina, and vulva. Cancer of the vagina is most common in older women. The cause is unknown, but proposed causes are chronic irritation with procidentia (uterine prolapse), wearing pessaries, genital viruses, or metastasis from another site. Cancer of the vagina is increasing in women under 50. The increased incidence in younger women may be related to concomitant or history of HPV. Vaginal carcinoma is usually asymptomatic, but the most common symptom is vaginal bleeding. Lesions of the cervix that are less than full thickness are called dysplasia. The lesions are categorized as mild, moderate, or severe. Bleeding (not itching) is the only significant symptom, but it does not necessarily occur early; cancers can be far advanced before they are discovered. Endometrium cancer can occur at any time during adulthood, but it is more common after menopause. Most carcinomas develop from a precancerous hyperplasia. Bleeding is the major symptom of endometrial carcinoma, indicating ulceration. The woman often notes a serous discharge before frank bleeding. Pain is a late symptom and indicates widespread disease. Ovarian tumors are found in postmenopausal women, ages 55-60. Neoplasms are most often asymptomatic resulting in late diagnosis. Symptoms are vague and typically occur after metastasis, such as a feeling of pelvic heaviness, urinary frequency, and changes in gastrointestinal functioning with feelings of abdominal fullness.

A 27-year-old woman presents in active labor. She is G3P2 and at 39 weeks of gestation. She has been receiving prenatal care since 6 weeks gestation, and her pregnancy has been uncomplicated. Both of her prior births were normal spontaneous vaginal deliveries. Her cervix is 6 cm, 90% effaced, mid-position, and soft. The fetus is not engaged and is thought to be vertex. Initial fetal monitoring shows a heart rate in the 140s with good accelerations, and it is reassuring. Contractions are 4 minutes apart, and she is comfortable. 20 minutes later, she experiences a large gush of clear fluid, and severe variable decelerations appear on the fetal heart rate monitor. Question What is the most likely diagnosis? 1 Cord prolapse 2 Placental abruption 3 Placenta previa 4 Uterine rupture

Correct answer: Cord prolapse Explanation The most likely diagnosis is cord prolapse. Cord prolapse occurs in 2 out of 1000 deliveries. It is diagnosed when fetal monitor recordings show that severe variable decelerations or bradycardia occur after membrane rupture. The cord is often palpable in the vagina. Cord prolapse happens most often at 5 cm cervical dilatation and in non-vertex presentations. 3 types of prolapse can occur: Overt cord prolapse is diagnosed when the membranes are ruptured and the umbilical cord falls through the cervix into the vagina ahead of the fetal presenting part. Funic presentation describes loops of umbilical cord between the presenting part and the cervical os prior to rupture of membranes. Occult cord prolapse is diagnosed when the cord is palpable alongside the presenting part on digital cervical exam. Immediate delivery is essential to prevent fetal compromise. Cesarean delivery is generally preferred when the cervix is not fully dilated. Vacuum or forceps delivery may be attempted if the cervix is completely dilated, although manual elevation of the fetal part and emergent cesarean are the most common management. Uterine rupture is less likely since the patient has no history of cesarean delivery. Abruption is often associated with tetanic contractions and bleeding. An increase in uterine tone is seen between contractions. Vasa previa usually presents with painless bleeding at rupture of membranes. Cervical exam would show fetal blood vessels covering the internal cervical os. Placenta previa typically presents with painless vaginal bleeding in the second half of pregnancy. An ultrasound should be done to confirm diagnosis, as digital rectal exam is contraindicated and can result in severe bleeding.

Case A 53-year-old woman is seen by her gynecologist. She has had three children and had her tubes tied through tubal ligation. She reached menopause at 48. Over the last few weeks, she has noted some vaginal bleeding that occurs unpredictably. She has had regular pap smears that have always been normal. The last one was 11 months ago. Speculum examination and bimanual palpation of the genitalia reveal no abnormalities. Question What is the next step in the management of this patient? 1 Repeat pap smear 2 Chlamydia serology 3 Laparoscopy 4 Endometrial biopsy 5 Hormonal profile

Endometrial biopsy Explanation Vaginal bleeding in a postmenopausal woman mandates endometrial sampling, with a view to ruling out endometrial cancer. Histopathological evaluation of material is essential. The most common causes of postmenopausal bleeding are vaginal atrophy (60-80%), exogenous estrogens (15-25%), endometrial cancer (10%), endometrial hypertrophy (5-10%), as well as cervical and endometrial polyps (2-12%). Individual clinicians may choose to order biopsy or ultrasound first, likely based on training and availability. This type of biopsy is collected with suction, not a scalpel, and done in the office in a matter of minutes. Ultrasound would typically require a separate scheduled appointment and would still require a subsequent biopsy for cytology if endometrial lining >5 mm. The patient had a Pap smear done less than a year ago that was normal. In light of her previously normal pap smears, the next one is not due for another 3-5 years. Pap smears may show the occasional dysplastic cell of endometrial origin, but it is by no means an accurate means of diagnosing endometrial pathology. Chlamydia serology is done for the diagnosis of pelvic discharge in sexually active women, not in cases of postmenopausal bleeding. Laparoscopy is not useful in the workup of postmenopausal bleeding. Exogenous estrogen (hormone profile) is a possible cause of postmenopausal bleeding, but the diagnosis is made by taking a careful history.

Case A 16-year-old girl presents due to concerns of significant acne and hirsutism. She states she is extremely embarrassed regarding this. Review of systems is positive for irregular periods. On exam, she is 5'2" tall and weighs 300 lb. You note a dark velvety discoloration of her neck fold. Question What laboratory finding would support the most likely diagnosis? Answer Choices 1 Low DHEA level 2 Elevated TSH level 3 Elevated LH: FSH ratio 4 Reduced testosterone level 5 Reduced insulin level

Elevated LH: FSH ratio Explanation Elevated LH:FSH ratio would support the diagnosis of polycystic ovary syndrome, marked by hyperandrogenism and anovulation. Patients often present with disordered menstrual cycles, hirsutism, and acne, which are related to elevated testosterone levels. About half of patients are overweight, related to insulin resistance. This patient also has evidence of acanthosis nigricans, which can be found in patients with insulin resistance. Typical lab findings of polycystic ovarian syndrome include increased androgen levels, including testosterone and DHEA. The LH:FSH ratio is typically elevated (>2). Insulin levels are high, related to insulin resistance. Elevated TSH level is consistent with hypothyroidism, which can cause weight gain, irregular periods, fatigue, cold intolerance, dry skin, hair loss, constipation, and many other symptoms. This patient denies any other symptoms consistent with hypothyroidism.

Case A 36-year-old woman presents with lower pelvic pain. She noticed that pain worsens during intercourse. She also reports heavy menstruation for the past 4 months. She has lost weight over the past few months. Upon physical exam, her gynecologist did a pelvic exam and felt a mass, but cervical motion tenderness and suprapubic tenderness was negative. A transvaginal ultrasound showed an increase of >4 mm of endometrial thickness. A D&C was then conducted and positive for dysplasia cells. Upon laboratory findings, the patient was negative for STDs and Escherichia coli. Question What is the most likely diagnosis? Answer Choices 1 Pelvic inflammatory disease 2 Urinary tract infection 3 Endometrial cancer 4 Urethritis 5 Pyelonephritis

Endometrial cancer Explanation A patient presenting with pelvic pain and heavy bleeding during menstruation with a mass on pelvic exam and endometrial thickness >4 mm upon a transvaginal ultrasound suggests endometrial cancer. With pelvic inflammatory disease, a mass would not be found on physical exam and cervical motion tenderness would be positive upon physical exam. A patient with a urinary tract infection presents with suprapubic tenderness; a mass would not be present with a UTI. Urethritis is most commonly caused by N. gonorrhoeae. The patient does not have any STDs. The patient is negative for Escherichia coli, which most commonly causes pyelonephritis.

Case A 27-year-old G1P1 woman has recently given birth by cesarean section to a 36-week male newborn. She did well throughout her pregnancy until 34 weeks gestation; she presented with fever, abdominal pain, and wetness. She was diagnosed with preterm premature rupture of membranes (PPROM) and chorioamnionitis. She was treated with steroids and antibiotics, stabilized, and then delivered by C-section with no complications. On postpartum day 7, she presents with sore breasts from breastfeeding and a sore abdomen. She admits to an odorous vaginal discharge, but she denies any associated vaginal bleeding. On physical examination, she has moderate lochia alba, and her temperature is 101.2°F. Question What is the most likely diagnosis? Answer Choices 1 Deep vein thrombosis 2 Endometritis 3 Infected retained placenta 4 Mastitis 5 Septic pelvic thrombophlebitis

EndometritisExplanation Prolonged rupture of membranes, cesarean delivery, prolonged labor, and multiple cervical examinations are all risk factors for postpartum endometritis. The presence of intra-amniotic infection increases the risk. Antibiotics are not routinely continued for chorioamnionitis after a delivery because the "source" of the infection (the placenta) has been removed. Whenever fever occurs in the immediate postpartum period, endometritis should be suspected. Mastitis is characterized by a swollen, firm, tender breast with systemic symptoms including inflamed breast, fevers, chills, and flu-like symptoms. Staphylococcus aureus is the typical pathogen. In the immediate postpartum period, however, breast engorgement without infection is the most likely reason for the patient's sore breasts. Pregnancy and the postpartum period increase the risk of thrombogenesis. This is highest the first few weeks postpartum, but DVT is not a likely source of the fever. Septic pelvic thrombophlebitis is a diagnosis of exclusion and is usually entertained when fever spikes continue following treatment for endometritis. Infected retained placenta is unlikely with no vaginal bleeding.

Case A 48-year-old Caucasian woman presents due to feeling like she is losing her mind. She wants some tests done. Upon further questioning, she reports she is having multiple episodes daily in which she suddenly becomes very hot, flushed, and diaphoretic. These episodes last about 1 minute, then resolve. She has not measured a fever. The patient reports that these episodes occur during the day and at night, the latter causing her to awaken drenched in sweat. As a result, her sleep has been poor, and she feels fatigued and irritable both at work and at home. She has noticed these symptoms for about the last 2 months, and they seem to be increasing in severity. The patient has not had a period for 3 months; she recently did a home pregnancy test, which was negative. Prior to that, she had regular menses. This patient denies weight changes, palpitations, cold intolerance, bowel changes, as well as changes in her nails, skin, and hair. Although she admits irritability, she denies anxiety, depressed mood, and suicidal ideation. Her past medical history is significant for seasonal allergies, which are relieved with over-the-counter antihistamines and taken as needed. Her surgical history includes a tonsillectomy and bilateral tubal ligation. Her family history is remarkable for diabetes in her maternal grandfather and hypertension in her father. She is a G4P3Ab1. Social history reveals the patient is an office manager for a dental clinic; she is a married nonsmoker with 3 children living at home. She denies any major psychosocial stressors recently. She drinks alcohol rarely, and she denies use of other drugs. Vitals and a urine specimen for hCG were obtained prior to the physical exam. Weight168 lbHeight65"Pulse72Blood pressure120/82Temperature98.2°FUrine hCGNegative Complete screening physical exam is normal, with normal sexual development and absence of hirsutism and acne. Question After discussing the likely diagnosis with you, the patient decides she does not want any tests now and wants to start treatment immediately. What medicine should be initiated? Answer Choices 1 Alprazolam (Xanax) 0.5 mg by mouth 3 times daily as needed 2 Clonidine (Catapres) 0.1 mg by mouth 2 times daily 3 Estradiol 1 mg by mouth daily 4 Estradiol/norethindrone acetate (Activella) 1 mg/0.5 mg by mouth daily 5 Estradiol vaginal (Estrace Vaginal) 0.01% Cream 1 g vaginally twice weekly

Estradiol/norethindrone acetate (Activella) 1 mg/0.5 mg by mouth daily Explanation The most effective treatment for vasomotor symptoms in perimenopause and menopause is hormone therapy. Because this patient still has her uterus, a combination of an estrogen plus a progestin should be selected. Estradiol/norethindrone acetate (Activella) 1 mg/0.5 mg daily is the most appropriate choice listed. This patient's history reveals no contraindications to the use of hormone therapy. Alprazolam would be given for panic attacks. This patient's described episodes are consistent with hot flashes and night sweats, and she denies anxiety. Alprazolam will not effectively treat vasomotor symptoms. Clonidine is an alpha-2 receptor stimulator and is used primarily for treatment of hypertension. It has mild efficacy for vasomotor symptoms and may be reasonable if there are contraindications to the use of the more effective hormone therapy. Estradiol may alleviate this patient's vasomotor symptoms, but without an "opposing" progesterone, it is contraindicated. Estrogens by themselves can overstimulate the endometrial lining, leading to endometrial hyperplasia and cancer. Vaginal use of estradiol, such as in Estrace Vaginal, is useful for treatment of vulvar and vaginal atrophy associated with menopause, but the majority of the effect is local with topical use of vaginal cream. This choice would not be effective in alleviating this patient's hot flashes and night sweats. She does not have symptoms or a physical exam consistent with vaginal atrophy (vaginal dryness/dyspareunia or pale, dry mucosa, respectively).

Case A 19-year-old woman presents with vaginal pruritus, dysuria, and dyspareunia. On exam, a profuse frothy yellowish-green vaginal discharge with reddened vaginal mucosa is noted. Question What is the next step in formulating a diagnosis? 1 Evaluate vaginal discharge under microscopy. 2 Perform a Pap smear. 3 Instruct patient how to douche. 4 Perform a vaginal ultrasound. 5 Perform a colposcopy.

Evaluate vaginal discharge under microscopy. Explanation A sample of the vaginal discharge should be evaluated under microscopy, particularly for the presence of trichomonads. Yellowish-green or gray frothy vaginal discharge is a classic finding in trichomoniasis, along with the accompanying signs and symptoms of pruritus, dysuria, and dyspareunia, which is due to the inflammation of the vaginal mucosa resulting from an acute infection. Although trichomonads are sometimes found on a routine Pap smear, it should not be used in diagnosing trichomoniasis. Colposcopy and vaginal ultrasound would not be recommended in diagnosing the most likely pathology. Patients should be discouraged from douching; doing so changes the vaginal flora.

Case A 46-year-old woman presents because she has not had her period for almost a year, has hot flashes, and feels fatigued most of the time. She complains of insomnia, and states that she keeps smoking despite trying many times to quit. Her libido is low and she does not use any hormone-based birth control. Her blood analysis indicates a TSH (thyroid-stimulating hormone) level of 3.1 mcU/mL and a FSH (follicle-stimulating hormone) level of 55.3 mIU/mL. Question What is the most accurate conclusion regarding the menstrual status of this patient? Answer Choices 1 TSH level is consistent with menopause. 2 FSH level is consistent with menopause. 3 Blood results are consistent with primary ovarian insufficiency. 4 TSH is consistent with hypothyroidism. 5 TSH is consistent with perimenopause.

FSH level is consistent with menopause. Explanation Based on the symptoms and blood analysis of the patient, the most likely diagnosis for this patient is menopause; she has not had a period for almost 12 months and her elevated FSH level is consistent with menopause. There is no known correlation between the TSH level and menopause or the menopause transition. Primary ovarian insufficiency is defined as a failure of ovarian function in women under 40. This leads to menopause-like symptoms, irregular or absent periods, and infertility, but the FSH level is not elevated (higher than 40 mIU/mL), as is the case in menopause. The patient's TSH does not indicate a deficiency or hypothyroidism of any kind.

Case A 35-year-old woman presents with a single firm, well-delineated, round, non-tender nodule in her left upper breast. It is very mobile with respect to its surrounding tissue. Question What is the most likely etiology of this mass? 1 Fibroadenoma 2 Breast carcinoma 3 Breast cyst 4 Fat necrosis 5 Tuberculosis

Fibroadenoma Explanation Fibroadenomas are benign lesions of the breast that usually occur before middle age. The following table describes the characteristics of fibroadenomas, breast carcinomas, breast cysts, fat necrosis, and tuberculous lesions. Other breast masses include galactoceles, which occur during or after lactation acute mastitis, which is generally hot, swollen, painful, indurated, and erythematous breast abscesses, which are hot, swollen, painful, fluctuant, and erythematous

Case A 16-year-old girl presents with a mass palpated over the left breast. There are no associated signs or symptoms noted. Menarche was at age 12; it is described as regular with moderate flow, lasting 3-5 days with occasional dysmenorrhea. She is in high school with good school performance. Vital signs are within normal limits. Weight and height are appropriate for age. Physical examination reveals a non-tender mass on the upper outer part of the left breast measuring about 2 x 3 cm. Aspiration is done, but no fluid is aspirated. An excisional biopsy is contemplated. Question What is the most likely diagnosis? Answer Choices 1 Breast abscess 2 Fibroadenoma 3 Cystosarcoma phyllodes 4 Fibrocystic disease 5 Fat necrosis

Fibroadenoma Explanation The clinical picture suggests fibroadenoma, the most common benign solid tumor containing glandular and fibrous tissue. They are mobile and well-defined and may be multiple. Hormonal changes are thought to play a role, as it persists during reproductive ages and increases in size with pregnancy/estrogen therapy, then diminishes following menopause. Clinical diagnosis is appropriate, but an ultrasound or needle aspiration can be done for clarity. Observation is appropriate for lesions <5 cm, as they typically regress spontaneously. Breast abscess usually presents with a painful fluctuant palpable mass associated with a systemic symptom like fever. It usually develops in ~10% of women with mastitis and is commonly due to inadequate treatment. Staphylococcus aureus and Streptococcus are the most common etiologic agents. Management is with antibiotics/drainage if necessary. Cystosarcoma phyllodes is a low-grade malignancy that typically presents in adolescents as an asymmetric breast enlargement with firm mobile circumscribed mass. This tumor can increase in size rapidly and become quite large. Excision is the treatment of choice. Although rare, malignant cases with metastases have also been reported. Fibrocystic changes form multiple small lumps that are usually tender with thickening of breast tissue, cystic formation (lumpiness), and nipple discharge. The cysts are usually due to blockage or dilatation of the ducts. Alleviate pain or discomfort with the use of soft brassiere and acetaminophen or NSAIDs. Reassure the patient that this is non-cancerous and typically resolves spontaneously over weeks/months. Fat necrosis in adolescents is caused by trauma and inflammation in the form of contusion or hematoma. These later result in cystic changes/fibrosis with retraction of the skin and nipple over the injured area. These changes may mimic malignancy, so biopsy may be the only means of differentiating them.

Case A 30-year-old woman presents with recurrent bilateral breast lumps and pain. She states that it seems to be worse during the last few days of her menstrual cycle; the lumps appear to get smaller after her cycle. She has felt different sized lumps in her breast that occur at the same time as the pain. On examination, several small, nodular lesions are noted in both breasts; they are freely movable. The axillary lymph nodes are unremarkable bilaterally. Question What is the most likely diagnosis? Answer Choices 1 Bowen's disease 2 Phyllodes tumor 3 Fibrocystic condition 4 Fibroadenoma 5 Fibrocarcinoma

Fibrocystic condition Explanation The clinical picture is suggestive of a fibrocystic condition; this occurs commonly in women 30-50. The patient may have an asymptomatic mass or may discover a mass while the pain is occurring. The pain usually worsens during the premenstrual cycle. Fluctuations in size and the disappearance of masses are common with this condition. Bowen's disease is a form of squamous cell carcinoma. It typically presents as well-demarcated erythematous plaques with scaling on the skin surface; they are typically seen in sun-exposed areas. Phyllodes tumor is a fibroadenoma-like tumor. These tumors can become quite large due to their rapid growth; they may recur after excision. They commonly occur in women 40-50. Fibroadenoma is a common benign neoplasm seen in young women. The fibroadenoma is described as round, rubbery, discrete, relatively mobile, and non-tender. Fibrocarcinoma is a malignant tumor of the breast. The tumor usually consists of a non-tender firm or hard mass with poorly defined margins.

Case A recently married 22-year-old woman presents with a 7-day history of severe vulval and vaginal pruritus and purulent vaginal discharge. She was having dysuria and dyspareunia. Examination showed frothy yellowish mucopurulent vaginal discharge with an offensive odor. Vaginal mucosa appeared inflamed, and cervical erosion was also observed. Vaginal secretion was collected, and a microscopy of wet film done immediately. Oval pear-shaped organisms about the size of white blood cells with wobbling rotatory motility were observed among inflammatory cells. Gram stain was negative for Candida and clue cells. Based on the diagnosis, the patient and her husband were treated with metronidazole. Question What is a characteristic of the microbial agent causing her symptoms? 1 Flagellated protozoan 2 Produces cystic form under unfavorable conditions 3 Able to survive on fomite for 2-3 days 4 Undulating membrane extends the full length of the parasite 5 Ideal pH for growth is 3.8-4

Flagellated protozoan Explanation The microorganism causing vaginitis in the patient is Trichomonas vaginalis, a flagellate protozoan parasite. For T. vaginalis, no cyst form is known. The parasite has four anterior flagella and a fifth flagellum along the undulating membrane. Flagella are the organs of motility. Cilia are absent. The undulating membrane of T. vaginalis is short and reaches up to the middle of the body, a differentiating feature from other trichomonads of humans in which the undulating membrane extends the full length of the parasite. The organism grows best under anaerobic conditions at 35-37°C, with an optimal pH 5.5-6. Normal acidic pH of 3.8-4 is detrimental to the growth of T. vaginalis. The trophozoite, a protozoan in the metabolically active growth stage, cannot survive outside the body for long, so transmission has to be from person to person by close contact. Humans are the only natural hosts of the parasite. T. vaginalis lives mainly in the vagina and cervix in women and the anterior urethra of men. Sexual transmission is the typical mode of infection. The parasite divides by longitudinal binary fission. Infection is most common in sexually active women of reproductive age. The incubation period ranges from 4 days to 4 weeks. Infection may be asymptomatic or cause acute inflammatory disease of the vagina and cervix. Laboratory diagnostic methods include microscopy, culture, antigen detection tests by enzyme-linked immunosorbent assay (ELISA), and molecular tests based on polymerase chain reaction (PCR). Microscopy for motile trichomonas in wet preparation of vaginal discharge should be performed within 10-20 minutes of collection to prevent the organisms from losing their viability. Nucleus, flagella, undulating membrane, and axostyle are prominent structures in stained smears. The CDC recommends metronidazole 2 g orally as a single dose for treatment of T. vaginalis infection. Treatment of sexual partners is also recommended. Metronidazole-resistant infections have been reported, and Tinidazole, a 5-nitroimidazole, is useful for treatment of such cases.

Case A 14-year-old girl presents to her family practice physician assistant with her mother. She reports no issues, has no problems at school, participates in school sports activities, and is not sexually active. Her past medical history is non-contributory. Her mother is worried because she still looks prepubertal and has not gotten her period (the mother had menarche at age 12). Height is 5 ft, weight is 79 lb (BMI 15.46; 3rd percentile), Tanner stage is 1 for breast and pubic hair development. The rest of physical examination is normal, including pelvic exam. Laboratory results are all within reference ranges (CBC, ESR, LFT, basic metabolic panel, and urine HCG). Question What hormone should be evaluated in this patient? Answer Choices 1 Adrenocorticotropic hormone (ACTH) 2 Estrogen 3 Follicle-stimulating hormone (FSH) 4 Progesterone 5 T4

Follicle-stimulating hormone (FSH) Explanation This patient is being evaluated for primary amenorrhea (absence of menarche by age 15). There is a lack of breast development. Follicle-stimulating hormone (FSH) is a pituitary hormone essential to test when evaluating causes of primary amenorrhea. Along with physical exam, it will help determine if the problem is in the hypothalamic-pituitary-gonadal axis (low or normal FSH) or in the ovaries (high FSH). ACTH is a pituitary hormone that regulates secretion of glucocorticoids by the adrenal glands. Disorders involving ACTH secretion include Cushing syndrome and Addison's disease. Neither of these disease processes is a cause of amenorrhea. Estrogen is regulated by FSH and is important for reproductive development and menorrhea, but it is not a cause nor an initial test used in diagnosing amenorrhea. Progesterone is regulated by luteinizing hormone (LH). Progesterone is essential to a healthy menstrual cycle, and levels are affected by amenorrhea, but it is not a cause of amenorrhea and not part of initial testing. A progesterone challenge may be administered to induce menorrhea in diagnosing secondary amenorrhea causes. T4 is regulated by thyroid-stimulating hormone (TSH). TSH is part of an initial secondary amenorrhea laboratory workup, since hyperthyroidism and hypothyroidism can cause menstrual abnormalities, but neither condition causes reproductive developmental delays.

Case A full-term female newborn has facial defects affecting the eyes, nose, and upper lips. She is the first child of non-consanguineous parents. The mother has a history of gestational diabetes, which began at the start of pregnancy. She contracted German measles a month before delivery. During the course of infection, her self-prescribed daily treatment was 3 tablets of aspirin, at least 6 cups of herbal tea, and a double dose of folic acid. Imaging studies showed prosencephaly. Question What risk factor is responsible for the condition of this newborn? Answer Choices 1 Rubella 2 Aspirin 3 Herbal tea 4 Gestational diabetes 5 Folic acid

Gestational diabetes Explanation The only possible risk factor during the period of the development of face and forebrain is gestational diabetes because the forebrain is formed and the face begins to develop in the fifth and sixth weeks of human pregnancy. Other known risk factors for prosencephaly include transplacental infections, bleeding during the first trimester, a history of miscarriages, use of some drugs potentially unsafe in pregnancy (insulin, birth control pills, lithium, anticonvulsants, retinoic acid, cholesterol-lowering agents, and maternal hypocholesterolemia), and toxins (alcohol, nicotine). Pregestational diabetes occurs when the mother has type 1 or 2 diabetes before pregnancy. Gestational diabetes can occur at any time during pregnancy. Gestational diabetes mellitus (GDM) is defined as glucose intolerance of variable degree with onset or first recognition during pregnancy. Rubella (German measles) is not likely the cause of birth defects because infection occurred after organogenesis was finished. Rubella can cause miscarriage, stillbirth, or birth defects (most often deafness, brain damage, heart defects, and cataracts) if infection occurs during the first 16 weeks of pregnancy. Aspirin taken during the last trimester may increase the risk of bleeding in the newborn, but cannot cause birth defects. Herbal tea is not the cause of birth defects after organogenesis is finished. All women capable of pregnancy should take a daily vitamin supplement of folic acid to prevent neural tube defects. In the late pregnancy, folic acid can neither prevent nor cause birth defects of the brain and face.

Case A 31-year-old multigravida known to have blood group A and Rh-negative red blood cells is pregnant with her third child. Her husband is also type A, but he is Rh-positive. She has an indirect Coombs titer at 1:16 dilution of her serum at 28 weeks gestation. Her past medical history includes two pregnancies; her first child (a boy) was healthy, while the second child (a girl) was born at 36 weeks gestation after the mother was noted to have an indirect Coombs titer at 1:16. Amniotic fluid obtained at 26, 28, 30, 32, and 34 weeks of gestation was analyzed by determining the optical density (OD) for bilirubin and indicated a progressive increase in the bilirubin level. Question What is the most likely explanation for the increase in bilirubin? Answer Choices 1 Hemolysis of red blood cells in the fetus 2 High red blood cell count 3 Increased maternal IgG 4 Rh factor is present in the fetal blood 5 Swelling of the fetus

Hemolysis of red blood cells in the fetus Explanation An increased level of bilirubin is most likely the result of the breakdown of heme, indicating that the fetus's red blood cells were hemolyzed. The fetus most likely has from HDN (hemolytic disease of the newborn). An increased production of red blood cells is associated with HDN. The fetus tries to compensate from the red cell hemolysis by producing more of them quickly. Most of the newly produced cells are immature and cannot function properly. As a result, the fetus can be severely anemic. The hematocrit is simply measured by centrifugation of unclotted blood and cell pellet volume assessment as a percentage of total blood volume, however, not by spectrophotometry. In HDN, increased hemolysis of the Rh-positive fetal red blood cells is caused by the maternal IgG antibodies that cross the placenta in increasing amounts during the second trimester of pregnancy, but IgG and Rh are both proteins, and they are usually detected by interactions with specific antibodies, not by changes in the optical density of the amniotic fluid. Swelling of the fetus is not correct. In HDN, if the fetus cannot cope with severe anemia, its heart can fail while fluid accumulates in the organs causing swelling. Such anomalies of the fetus are usually detected by ultrasonic visualization, however, not by optical density changes of amniotic fluid.

Case A 24-year-old African American G2P1 presents to her obstetrician at 34 weeks gestation with weight gain, fatigue, diffuse "swelling," and headache. On physical examination, you note periorbital edema and 3+ pitting edema in the lower extremities peripherally. Physical examination also demonstrates tenderness to palpation in the right upper quadrant. The patient has increased reflexes bilaterally. Vital signs are as follows: Temperature is 99.2°F. Pulse is 89 beats/minute. Respirations are 18. Blood pressure is 174/99 mm Hg. Urinalysis reveals no erythrocytes per high power field and no casts, but there is a large amount of protein in the urine. Laboratory values are as follows: Question What is the most likely diagnosis? 1 Disseminated intravascular coagulation (DIC) 2 Hemolysis, elevated liver tests, and low platelets syndrome (HELLP) 3 Idiopathic thrombocytopenic purpura (ITP) 4 Thrombotic thrombocytopenic purpura (TTP) 5 Normal laboratory values of pregnancy

Hemolysis, elevated liver tests, and low platelets syndrome (HELLP) Explanation These laboratory findings are not normal, and the clinical history and low platelets are most consistent with HELLP syndrome. HELLP Syndrome is most often associated with third-trimester pre-eclampsia, HTN, and proteinuria. The acronym HELLP stands for Hemolysis, Elevated Liver Tests, and Low Platelets. While most patients with pre-eclampsia can be managed expectantly, patients with HELLP need to be induced for delivery (based on the maturity of the fetus), given the high risks associated with this condition. Patients with HELLP syndrome also need to be given magnesium sulfate to prevent seizure activity. Disseminated intravascular coagulation (DIC) can occur as a consequence of HELLP syndrome, placental abruption, sepsis, amniotic fluid embolism, or intrauterine fetal demise. Patients with DIC have clinical manifestations, including epistaxis, hematuria, purpura, and bleeding from puncture sites or incision sites. Fibrinolysis is an important component of DIC, and laboratory values show elevated levels of D-dimer and fibrin degradation products (FDPs). Platelet counts are usually low; clotting times (aPTT and PT) are elevated; antithrombin levels and individual clotting factors (V and VII) may be diminished. Given the lack of associated risk factors, DIC would be an unusual diagnosis in this patient. Idiopathic thrombocytopenic purpura (ITP) is diagnosis of exclusion and reflects a disease of increased peripheral platelet destruction, as most patients have antibodies to specific platelet membrane glycoproteins. Patients usually have physical findings of thrombocytopenia, including purpura, increased bruising tendencies, and epistaxis. In this patient with known ITP, high-dose parenteral glucocorticoids and IV immunoglobulin (IVIG), with or without platelet transfusions, are appropriate treatment. Splenectomy is usually reserved for patients when medical therapy fails. Thrombotic thrombocytopenic purpura (TTP) is the multi-system condition characterized by microangiopathic hemolysis and platelet aggregation with hyaline thrombi that forms unrelated to coagulation system activity. In TTP, platelet microthrombi predominate, forming in systemic arterioles and capillaries, leading to partial occlusion of these small vessels. Organ ischemia, thrombocytopenia, and erythrocyte fragmentation ensue. Additionally, most patients have altered mental status, including confusion, generalized headaches, focal deficits, seizures, visual disturbances, and/or coma. Renal insufficiency and gross hematuria may also be seen.

A 52-year-old woman presents because her menopausal symptoms have been extremely distressing. Over the past 4 months, she has experienced severe mood swings, hot flashes, night sweats, breast tenderness, and changes in her appetite. She has never smoked; she has an occasional drink. She had an IUD that was removed at age 35. She had irregular periods in her 40s and menses stopped at 50. There is no family history of cancer. After a prolonged discussion, a decision to start hormone replacement therapy is made. Question What is a concern with the use of an estrogen-only supplement for this patient? Answer Choices 1 Vaginal dryness and itching 2 Worsening mood swings 3 Lowering of serum HDL level 4 Higher risk of vertebral fractures 5 Higher risk of endometrial cancer

Higher risk of endometrial cancer Explanation When estrogen is administered as the sole therapy without cyclical progesterone, it causes unopposed proliferation of the endometrium. Over a period of time, this increases the chance of dysplasia, leading to endometrial cancer. To prevent this, estrogen is usually administered with cyclical progesterone. Estrogen helps to maintain the epithelial lining of the vagina and increases lubrication, decreasing the symptoms of postmenopausal atrophic vaginitis (vaginal dryness and itching). Hormone replacement therapy during menopause is beneficial in mitigating mood symptoms. When used, they are prescribed in the lowest dose possible for the shortest period of time. Estrogen raises HDL and lowers LDL levels. It acts as a cardioprotective agent in that respect. Estrogen influences bone metabolism and prevents osteoporosis.

Case A mother brings her 16-year-old son to your medical office for a comprehensive history and physical examination. She tells you she is concerned about his immature physical development and insecure behavior. She thinks these characteristics are markedly different from her other children. His IQ is 70, and he is in special education for a language-based learning disability. On physical examination, he is tall and thin; he has sparse body hair and a high-pitched voice. Heart, lungs, abdomen, and neurologic exam are unremarkable. Pertinent positive findings include disproportionately long arms and legs, gynecomastia, as well as small testes and phallus. Question What is the most likely diagnosis? Answer Choices 1 Fragile X syndrome 2 Klinefelter syndrome (XXY) 3 Turner syndrome (XO) 4 Triple X syndrome (XXX) 5 XYY syndrome (XYY)

Klinefelter syndrome (XXY) Explanation The combination of hypogonadism, long extremities, decreased intelligence, and behavioral problems makes Klinefelter syndrome (also referred to as XXY syndrome, 47,XXY, and Klinefelter's syndrome) the most likely diagnosis. The original syndrome, as described by Dr. Klinefelter, consisted of gynecomastia, testicular atrophy, and infertility. Intelligence profiles can range from specific learning disabilities (language learning or reading impairment most common) to intellectual disability. The only constant feature of the syndrome is testicular atrophy with resulting azoospermia and infertility. The atrophy of the testis is the result of fibrosis, which begins to appear in childhood and progresses until all the seminiferous tubules are replaced by fibrous tissue. In males presenting with gynecomastia, intellectual disability, and eunuchoidism (loss of male secondary sexual characteristics, small penis, loss of body hair, and a high-pitched voice), Klinefelter syndrome should be at the top of the list in the differential diagnosis. Most patients with Klinefelter's syndrome have 47 chromosomes instead of the normal 46 chromosome karyotype. The extra chromosome is an X chromosome, making the sex chromosome constitution XXY instead of XY. Klinefelter's syndrome is one of the most common chromosome abnormalities seen in males and occurs in 1 in 300 of the male population. Patients with this syndrome show that the Y chromosome is strongly sex-determining; a patient who has an XXY chromosome constitution may therefore have the appearance of a normal male—with infertility being the only incapacity—while the loss of a Y chromosome leads to the development of a bodily form that is essentially feminine. Fragile X syndrome is characterized by prominent jaw, large ears with soft cartilage, and macroorchidism in pubertal male patients. Turner syndrome (XO) is a genetic condition of females patients usually characterized by a short stature, increased distance between the nipples, low hairline, low set ears, a webbed neck, amenorrhea, and sterility. Triple X syndrome (XXX) only occurs in female patients. In XYY syndrome (XYY), IQ is normal, and there is normal sexual development and normal fertility.

Case A 27-year-old woman presents with cramping abdominal pain and vaginal bleeding. Further history reveals amenorrhea for the past two menstrual cycles. On examination, she is found to have left lower abdominal tenderness and an adnexal mass. Lab values reveal an elevated beta HCG level. Question A further history would possibly reveal what associated risk factor? 1 History of PID 2 Diaphragm use 3 Use of condoms 4 Oral contraceptive pills 5 Vaginal spermicides

History of PID Explanation The patient has a potential ectopic pregnancy, evidenced by the triad of amenorrhea, adnexal mass, and an elevated B-HCG. Inflammation of the fallopian tubes secondary to bacterial infection resolves with fibrosis and adhesion formation. The fertilized egg gets trapped on its transit through the fallopian tube, resulting in an ectopic pregnancy. Being thin-walled, implantation in the fallopian tube invariably results in a rupture. Diaphragms sit atop the cervix, preventing sperm from entering the uterine cavity. They are used as a temporary measure and are not associated with ectopic implantation. Condom use is a temporary measure not associated with ectopic implantation. Regular OCP use is associated with an extremely low failure rate. In the event of contraceptive failure, there is no increased risk of ectopy with OCP use. Vaginal spermicide use appears to decrease a woman's risk of acquiring bacterial STDs, particularly C. trachomatis and N. gonorrhoeae cervical infections, the most common organisms in cases of PID. In addition, most women use a spermicide with a diaphragm, which may further decrease the risk for PID.

Case A 24-year-old woman presents with a 3-month history of excreting fluid from her left breast. When her symptoms first started, she had noted clear fluid staining her bra, which recurred infrequently. If she compressed the left breast tissue around her nipple area, she could easily express clear fluid. Occasionally, with difficulty, she was also able to express the same fluid from her right breast. Medical and surgical history is positive for depressive disorder; it has treated with tricyclic antidepressants for the past 4 months; she had an appendectomy 2 years ago. Her menstrual history is regular, with dysmenorrhea present. She drinks alcohol occasionally. Family history is positive for diabetes. Question What history has the most significance in regard to this patient's condition? 1 History of tricyclic antidepressant use 2 Past history of appendectomy 3 Family history of diabetes 4 Menstrual history 5 Personal history of alcohol intake

History of tricyclic antidepressant use Explanation This patient seems to have galactorrhea. Causes of abnormal lactation can be broken down into the following categories: Any disruption of production of dopamine from the hypothalamus will remove the inhibition of dopamine on the pituitary gland; this will result in increased prolactin production. Thus, craniopharyngiomas, empty sella syndrome, and anti-dopamine drugs (like metoclopramide and antipsychotics) can all antagonize the normal production of dopamine from the hypothalamus. Fluctuating hormone levels of prolactin with a relative deficiency of estrogen and progesterone; therefore, pregnant women may lactate as early as the second trimester and continue to produce milk for up to 2 years after the cessation of breastfeeding. These fluctuating levels may also cause lactation during puberty and menopause. Similarly, stimulation of the breast (suckling, chest wall disorders, thoracic nerve irritation) and nipple tissue (poor fitting bras or irritating clothes) can increase the prolactin enough to stimulate lactation. Increased levels of TRH will stimulate prolactin production. Similarly, hypersecretion of cortisol and growth hormone may have associated hyperprolactinemia. Chronic renal failure will decrease the excretion of prolactin and lead to hyperprolactinemia. Serotonin also seems to play a part in increasing prolactin and patients on selective serotonin reuptake inhibitors (SSRIs, like fluoxetine) and cyclic antidepressants (like amitriptyline and clomipramine) have reported galactorrhea. Contraceptive hormones can cause galactorrhea; this side effect seems to occur more often after discontinuation of oral contraceptive pills than during their use (similar to the withdrawal effect of precipitous drop of estrogen and progesterone that occurs after delivery). Illicit drugs (amphetamines, cannabis, opiates, etc.) can all cause galactorrhea. Similarly, a number of herbs used in supplements have been reported to cause galactorrhea (fennel, anise, etc.). Pituitary tumors, such as a pituitary prolactinoma, can cause galactorrhea. About 20% of females with galactorrhea have evidence of a pituitary tumor on testing, but this rate increases to 34% if the woman has concomitant amenorrhea. Some cancers oversecrete prolactin (e.g., renal cell cancers). In summary, the causes of galactorrhea are: Physiologic conditions (14%) Neoplastic process (Benign or malignant 18%) Hypothalamic-pituitary disorders (non-prolactinoma/under <10%) Systemic disease (<10%) Medications and herbs (20%) Chest wall irritation (<10%) Idiopathic (35%) As can be seen from this breakdown, the most common cause of galactorrhea is idiopathic; this is a diagnosis of exclusion. The history must still be completed to rule out serious causes of galactorrhea. Tests to exclude hormone etiologies include prolactin level, human chorionic gonadotropin level, thyroid-stimulating hormone level, and BUN/Creatinine. In this patient, there is history of tricyclic antidepressant use, which points to the diagnosis of pharmacological galactorrhea. A menstrual history of amenorrhea, not dysmenorrhea, would be significant; a surgical history of a chest operation, not an appendectomy, would also be significant. A family history of thyroid disorder or multiple endocrine neoplasia type I, not diabetes, as well as illicit drug use, not alcohol intake, would have more significance.

Case You are evaluating a 26-year-old man; he is suspected of being infertile. His past medical history is unremarkable. On examination, you note he is 6'4"; he has mild gynecomastia, sparse body hair, and small soft testes. Question What is the most likely diagnosis? Answer Choices 1 Cystic fibrosis 2 Turner syndrome 3 Klinefelter syndrome 4 Fragile X syndrome 5 Hypospadias

Klinefelter syndrome Explanation Klinefelter syndrome is a sex chromosome disorder due to an additional X chromosome on a male karyotype: 47, XXY. Patients present either as adolescent boys who fail to progress through puberty or as adult males suspected of having primary infertility. Low androgen levels lead to gynecoid features, including a female pattern of body hair and gynecomastia. The testes are typically small and soft, and libido also may be reduced. Cystic fibrosis can cause male infertility due to absent development of the vas deferens. Tall body habitus, gynecomastia, and small testes are not typical of cystic fibrosis. Recurrent pulmonary infection and intestinal malabsorption are also features of cystic fibrosis; these are lacking in the described case. Turner syndrome is noted by a 45, X karyotype, short stature, webbed neck, and primary amenorrhea in women. Fragile X syndrome is due to a trinucleotide repeat expansion in the fragile X gene in boys/men. It is also characterized by learning difficulties and, in more severe cases, overt cognitive disability. The testes are typically enlarged. Hypospadias is a developmental problem of the penis where the urethral meatus is proximally placed. Most cases are sporadic, and fertility can be affected, but gynecomastia, altered body hair distribution, and small testes are not typically associated with hypospadias.

Case A 33-year-old G5P4 woman presents to her gynecologist. She has been using condoms and would like to discuss alternative birth control options. She has regular periods and is not currently trying to get pregnant. She is sexually active with one partner. Past medical history includes asthma, deep vein thrombosis during her first pregnancy, and a C-section for her fourth pregnancy. She has no known drug allergies. Question What is the best birth control option for this patient? Answer Choices 1 Combination estrogen-progestin pill 2 Ethinyl estradiol/norelgestromin patch 3 Levonorgestrel IUD 4 Tubal ligation 5 Total abdominal hysterectomy

Levonorgestrel IUD Explanation Mirena (levonorgestrel) IUD is the most appropriate choice. The patient has a history of DVT, so estrogen-containing therapy is contraindicated due to its elevated risk of recurrent thromboembolism. Mirena IUD contains progesterone and can be safely used by the patient. Combination estrogen-progestin pill and ethinyl estradiol/norelgestromin patches contain estrogen. Estrogen-containing products are contraindicated with history of DVT. The patient is not currently trying to get pregnant, but the history does not mention if the patient desires future pregnancy, making tubal ligation inappropriate. Total abdominal hysterectomy is indicated when a vaginal or laparoscopic procedure is not clinically indicated. Indications for a hysterectomy include uterine fibroids and other benign tumors.

Case A 29-year-old woman is taking D-penicillamine for Wilson's disease. Today, her laboratory findings (including liver function tests) are within normal limits; there are no neurological signs. Her work requires frequent travel (3-5 days per week), including international overnight travel all over the world. She wants contraception that is both effective and convenient in terms of application. Question What is the best form of contraception for this patient? Answer Choices 1 Combined oral contraceptive pill 2 Progestin-only pill 3 Non-hormonal copper intrauterine device 4 Levonorgestrel-releasing intrauterine system 5 Contraceptive patch

Levonorgestrel-releasing intrauterine system Explanation Only spermicide, barrier contraceptives, and progesterone-only preparations should be considered in Wilson's disease. Levonorgestrel is a second-generation progestin (synthetic progestogen). A levonorgestrel-releasing intrauterine system causes thickening of cervical mucus, reducing sperm motility and penetration; over time, it decreases proliferation of the endometrium. The local inflammatory response could be toxic to sperm, providing an additional contraceptive effect. Combination oral contraceptive pills contain both estrogen and progestins. Estrogen-containing contraceptives are not recommended since estrogen may interfere with copper excretion. The progestogen-only pill is efficient, but it must be taken at or around the same time every day (within 3-12 hours). The pill may be inconvenient for a woman with a job that requires international travel all over the world. Non-hormonal copper intrauterine devices should be avoided in a disease that has the main characteristic of copper accumulation in the body. Copper IUDs primarily work by disrupting sperm mobility and damaging sperm. Contraceptive patches also contain estrogen, so they should be avoided in Wilson's disease.

Case 6 hours after delivery, a 25-year-old primigravida presents with fullness and pain in the lower abdomen. Her pregnancy was normal; labor was induced with misoprostol and progressed normally during the first stage; episiotomy was performed 2 hours after second stage had started; and the third stage of labor was normal. The newborn weighed 3800 g, and APGAR score was 9, 9, and 10 in 5 minute intervals. Postpartum visually estimated blood loss is about 550 mL (normally ≥500 mL in the first 24 hours after delivery). She did not void after the delivery. On examination, her vitals are stable and her bladder is overdistended. Question What is the next step in the management of this patient in addition to catheterization? Answer Choices 1 Measurement of urine volume 2 Antibiotics 3 Oxytocin 4 Uterine massage 5 Urodynamic studies

Measurement of urine volume Explanation This patient has overt urinary retention because of the inability to void spontaneously within 6 hours of vaginal birth and the physical finding of overdistended bladder. It is unlikely that she will void now if she did not void within first 4-6 hours after delivery. Postpartum urinary retention occurs in 10-15 % of women and is probably multifactorial. Risk factors in this case are the primiparity, prolonged and induced labor, and episiotomy. She should be catheterized, and the urine volume measured. Urine volume will guide further management: if >200 mL is obtained, the catheter should be left for another 24 hours. If <200 mL, the catheter may be removed and the bladder checked again. Antibiotics are not indicated because she has no signs of infection. Oxytocin is not indicated in a patient with stable vital signs when there are no signs and symptoms of atony of the uterus. Besides, oxytocin may cause rapid bladder filling, worsening the condition. Uterine massage may be considered in cases of postpartum hemorrhage. Urodynamics studies evaluate the functional status of the lower urinary tract, usually in the cases of urinary incontinence.

Case A 54-year-old woman presents for her annual pelvic examination. Her last menstrual period was 1 year ago, and her last few cycles were extremely irregular. She describes multiple daily episodes of severe, intense heat in the face and trunk accompanied by sweating. She states that these "heat episodes" have been occurring 4-6 times daily for 4 months, and they interfere with her everyday activities and sleep. She reports no other symptoms. She has received yearly annual pelvic examinations and clinical breast exams and mammograms without any significant findings. Her past medical history is negative for cardiovascular disease, blood clots, and breast cancer. Her pelvic examination has findings of excessive dryness and apparent vaginal wall atrophy. Question What pharmacologic intervention can assist in minimizing these symptoms? 1 Menopausal hormone therapy 2 Low-dose oral contraceptives 3 Phytoestrogens 4 Progestin-releasing intrauterine devices 5 Testosterone replacement therapy

Menopausal hormone therapy Explanation Menopause is diagnosed after 12 months of amenorrhea. Menopausal hormone therapy is considered a safe option for women who are healthy, symptomatic, and within 10 years of menopause or under 60. Contraindications include history of breast cancer, coronary artery disease, previous thromboembolic event or stroke, or active liver disease. Oral and transdermal preparations are available. Low-dose oral contraceptives are an option for perimenopausal women for relief of menopausal symptoms and for contraception and control of heavy menses. Phytoestrogens are substances found in plants such as soy and red clover extracts that bind to estrogen receptors. They are not a substitute for systemic estrogen and do not appear to significantly improve menopausal hot flashes. Combination therapy with estrogen and progesterone is recommended in women with an intact uterus (to prevent endometrial hyperplasia from unopposed estrogen). In clinical trials for sexual dysfunction in postmenopausal women, some benefit to testosterone therapy is suggested, but it cannot be routinely recommended until effects are better known.

Case A 50-year-old woman with no significant past medical history presents for an annual pelvic exam. Her last menstrual period was over 6 months ago, and the last few occurrences of menses were extremely irregular. The patient also describes having the sensation of intense heat in the face and trunk accompanied by sweating. She also states that her "heat episodes" have been occurring 1-2 times weekly for several months. She denies any other symptoms. She has received her annual pap and pelvic examination yearly and a clinical breast exam without any issues. On pelvic examination, you note obvious vaginal thinning, excessive dryness, and apparent vaginal wall atrophy. Question What is most likely diagnosis? Answer Choices 1 Ovarian failure 2 Menopause 3 Pregnancy 4 Cervical cancer 5 Polycystic ovary syndrome

Menopause Explanation Menopause is a cessation of menstruation due to natural aging or an external cause (surgical). There is usually a 1-3 year time period when women will typically adjust physiologically to the diminished hormonal and menstrual actions and their effects on the body: hot flashes, night sweats, vaginal dryness, and in the later stages, osteoporosis. In western societies, the average age at which a woman experiences menopause is 51. Ovarian failure (premature menopause) is the cessation of menses before age 40. Pregnancy is a possibility in this patient, but it is not as likely. Cervical cancer is not likely because this patient has obtained her yearly pelvic exams without any significant abnormalities. Polycystic ovary syndrome is another disease state that has a main symptom of amenorrhea, but it typically presents in younger women. She has never had this diagnosis before, and it is usually found in women in childbearing years who may be having infertility issues due to the hormonal dysfunction that occurs.

Case A 42-year-old woman has a 12-month history of amenorrhea. She noticed she has been having hot flashes and night sweats, loss of libido, and vaginal dryness. Upon physical exam, no abnormal findings are found except some dry skin. Upon lab results, elevated serum FSH level >30 mIU/mL. Her TSH and prolactin are normal and her serum HCG is negative. Her 24-hour urinary free cortisol is 11 μg/24 hr, within the normal range of 10-100 μg/24 hr. Her serum IGF-1 level is in the normal range for a 42-year-old (90-360 ng/mL). Her levels of ACTH are within normal limits. Question What is the most likely diagnosis? 1 Menopause 2 Polycystic ovary syndrome 3 Pituitary adenoma 4 Acromegaly 5 Cushing syndrome

Menopause Explanation Menopause is the most likely diagnosis due to her age, amenorrhea over 12 months, vaginal dryness, mood swings, night sweats, and lab results with the absence of other secondary causes. FSH is usually elevated. Polycystic ovary syndrome is not the most likely diagnosis, as FSH levels would be normal or low. This patient's 24-hour urinary free cortisol 11 μg/24 hr is within normal range with normal ACTH. Serum IGF-1 and prolactin are also normal, which can further support this patient not having an adenoma. This patient does not have visual changes, which is the most common presenting symptom. Her serum IGF-1 is within normal limits. If the patient had acromegaly, her serum IGF-1 would be twice the upper limit. A patient with Cushing syndrome would have an elevated level of ACTH and high ACTH levels.

Case A 15-year-old girl presents with a 3-day history excessive vaginal discharge without itching or burning. She denies ever being sexually active. Her last menstrual period was 10 days ago. On examination you find a thin, white, homogeneous discharge that has a distinct amine odor when potassium hydroxide is added. On saline wet mount, epithelial cells are covered with bacteria. Question What is the most appropriate management of this patient? Answer Choices 1 Metronidazole 2 Doxycycline 3 Ceftriaxone 4 Fluconazole 5 Azithromycin

Metronidazole Explanation This patient has the classic signs of bacterial vaginosis (BV), including the thin discharge with the fishy odor on KOH prep, and clue cells on wet prep. Standard treatment of BV is metronidazole 500 mg po bid for 7 days. BV is not sexually transmitted; rather, it most typically occurs from a shift in vaginal flora away from the Lactobacillus species toward more diverse bacterial species, including anaerobes. Gardnerella vaginalis is the most common pathogen leading to BV. This overgrowth of bacteria occurs due to a shift of the pH balance within the vaginal area (pH rises from the normal 4.0-4.5 for estrogenated women to >4.5). Menses can cause such pH shift. Doxycycline and azithromycin are prescribed to treat chlamydia. Fluconazole is used in the treatment of candidiasis. Ceftriaxone is given to treat gonorrhea.

Case A 25-year-old woman presents for an annual gynecological visit. Menses onset was at age 12; duration of menses is 6 days, every 30 days. She is nulliparous with no history of abnormal pap smears or diagnosed STIs. She does not smoke and has been in a monogamous relationship with 1 partner for the past year. The patient mentions a whitish-gray vaginal discharge that increases after intercourse, accompanied by a distinct musty odor; she denies pain from this discharge. Physical examination and a positive result of a whiff test support the suspected diagnosis. Question What is the most appropriate therapy? 1 Miconazole vaginal suppository 2 Metronidazole by mouth 3 Tinidazole by mouth 4 Fluconazole by mouth 5 Doxycycline by mouth

Metronidazole by mouth Explanation This patient most likely has cervicitis caused by bacterial vaginosis (BV), a polymicrobial disease state not viewed as a sexually transmitted disease. Its main etiology is an abundant overgrowth of Gardnerella and other anaerobes. These organisms produce a malodorous discharge with a characteristic grayish frothy appearance to it. An amine-like (fishy) odor is created if the discharge is alkalinized with 10% potassium hydroxide (positive whiff test). On wet mount, the characteristic clue cells will help the healthcare provider confirm the diagnosis. Appropriate treatment is metronidazole by mouth. A patient with a Candida infection will have a malodorous, white, curd-like discharge with extreme pruritus and vulvovaginal erythema. Microscopic examination with potassium hydroxide will reveal hyphae and spores. Miconazole and fluconazole are treatment options for a confirmed Candida infection. Trichomoniasis cervicitis (strawberry cervix) is diffuse vaginal erythema accompanied by red macular lesions and punctate hemorrhages on the cervix. It produces a malodorous, frothy, yellow-green discharge with pruritis. Microscopic examination will reveal motile protozoan organisms with flagella. Tinidazole is the treatment of choice; it also has a role in second-line therapy of BV. Doxycycline is part of the treatment regimen in pelvic inflammatory disease (salpingitis/endometritis). Signs and symptoms include lower abdominal pain, chills, fever, menses disturbances, purulent cervical discharge, and exquisite cervical and adnexal tenderness to palpation. This pathology is more common in nulliparous, young, sexually active women with a history of multiple partners.

Case A 26-year-old woman presents with 20 weeks of amenorrhea. This is the first time she has been to the doctor in 5 years, and she believes she is pregnant. You perform a complete physical examination and cannot auscultate the fetal heart rate. The ultrasound reports a "snowstorm" pattern with placental and fetal remnants missing. The patient tells you she has history of miscarriage "that had something to do with high blood pressure." The doctor explains that she is not pregnant and has a disease related to the proliferation of trophoblasts. Question What is a risk factor in the development of this disease? Answer Choices 1 Alcohol consumption 2 Eclampsia 3 Hyperthyroidism 4 Multiparity 5 Nutritional deficiency

Nutritional deficiency Explanation Nutritional deficiency is a risk factor in the development of hydatidiform moles. Deficiency of animal fats and carotene are to blame, especially vitamin A deficiency (carotene), folic acid deficiency, and low protein. Hydatidiform moles increase the proliferation of trophoblasts. Partial hydatidiform moles are diagnosed when there is a fetus (karyotype 69,XXY or 69,XXX). Complete hydatidiform moles are diagnosed when there is no fetus (karyotype 46,XX or 46,XY). Complete hydatidiform moles are more common than partial. Other risk factors include previous molar pregnancy and young (teenage) or advanced maternal age. Risk for mole development is 5-10 times greater if the patient is over 40. Alcohol consumption is not a known risk factor for molar pregnancy, but it can increase the risk of fetal alcohol syndrome (FAS), characterized by anatomic deformities and intellectual disability. Eclampsia is not a risk factor for developing a molar pregnancy. Once a molar pregnancy exists, however, it can be a risk factor for development of malignancy. 20% of complete hydatidiform moles become malignant. Hyperthyroidism, like eclampsia, increases the risk in malignancy once a mole exists, but it is not a risk factor in the initial development of a hydatidiform mole. Multiparity does not increase the risk of developing hydatidiform mole.

Case A 33-year-old woman presents with spider angiomas on her trunk and face. They appeared during pregnancy and have gotten worse in the 9 months since delivery. She takes imipramine for depression and resumed oral contraceptives after delivery. She had acute infection with hepatitis C virus 3 years ago. She was also frequently treated with flucloxacillin during the past few years for recurrent respiratory tract infections. Physical examination reveals spider angiomas on the patient's face, forearms, and back. Laboratory analyses reveal AST (26 IU/L), ALT (22 IU/L), and alkaline phosphatase (43 IU/L) levels within reference ranges. HBsAg, HBeAg, antiHbc antibodies, and HCV RNA are negative, and IgG antibodies are present. Question What is the most likely cause of the patient's spider angiomas? Answer Choices 1 Flucloxacillin-induced hepatitis 2 Imipramine-induced hepatitis 3 Oral contraceptives 4 Chronic hepatitis from hepatitis C virus 5 Imipramine use

Oral contraceptives. Explanation In conditions where estrogen levels are elevated (pregnancy, contraceptive intake, liver disease due to decreased degradation of estrogens), the surplus of estrogens causes the appearance of spider nevi. This patient's symptoms appeared during pregnancy and worsened with oral contraceptives. In pregnant women, spider angiomas appear from months 2-5 of pregnancy and disappear within 2 months of delivery. Since spider angiomas are present for longer in this patient, it is unlikely they are purely pregnancy-related. 80% of untreated patients with acute hepatitis C develop chronic infections; only about 20% of patients recover completely. Patients who have chronic hepatitis C also have HCV RNA in their serum, but those who have recovered completely have no HCV RNA in serum and have IgG antibodies against hepatitis C virus as the marker of past hepatitis C infection. This patient satisfies criteria for the patient who had hepatitis C in the past, so her spider angiomas are not related to chronic hepatitis C infection. Imipramine rarely causes liver damage. Hepatitis caused by imipramine administration is cholestatic—associated with increased serum levels of transaminases and alkaline phosphatase. Since the alkaline phosphatase level in this patient is within reference range, imipramine-induced hepatitis is unlikely. Similar to imipramine, hepatitis caused by flucloxacillin administration is cholestatic. Flucloxacillin-induced hepatitis is unlikely.

Case A 35-year-old woman presents with amenorrhea. She missed her last period and reports fatigue, morning episodes of nausea and vomiting, dry eyes, and difficulty seeing at night. Physical examination reveals dry conjunctiva, corkscrew hair, and hyperkeratotic skin lesions. Laboratory evaluation is remarkable for a significantly elevated serum hCG. Transvaginal ultrasound reveals a molar pregnancy. Question A blood test will most likely indicate low levels of what compound? Answer Choices 1 Folic acid 2 Iron 3 Protein 4 Retinol 5 Ascorbic acid

Retinol Explanation This patient's symptoms can be associated with retinol (vitamin A) deficiency, which is associated with increased incidence of complete molar pregnancies, night blindness, poor bone growth, chronically dry conjunctiva, and hyperkeratotic skin lesions. These clinical symptoms are not consistent with folic acid, iron, protein, or ascorbic acid (vitamin C) deficiency. None of these deficiencies are associated development of molar pregnancies.

Case A previously healthy 30-year-old woman G1P1 presents with amenorrhea, weight loss, shortness of breath, and increasing abdominal circumference. Menstrual irregularity started about 1 year ago, and her last menstruation was 3 months ago. She has lost around 5 kg over the last few months, but her waist has enlarged. Shortness of breath started a week ago, and it is worse when she is lying down; she now sleeps using at least two pillows. An examination of the lungs shows dullness to percussion, decreased tactile fremitus, and inaudible breath sounds bilaterally. Physical examination of the abdomen shows bulging of the flanks in the reclining position, and there is a difference in percussion in the flanks that shifts when she turns on her side. Pelvic examination shows a normal uterus and left adnexa; the right adnexum appears enlarged, smooth, and tender. Complete blood count and chemistry is normal, and a chest X-ray confirms the presence of pleural effusion on the right side. Fluid obtained from peritoneal cavity shows heterogeneously bloody content that clots; the leukocyte number is normal, and serum-ascites albumin gradient (SAAG) is 0.8 g/dL (low). The sample is negative of malignancy. Question What is the most likely cause of her symptoms? 1 Extrauterine pregnancy 2 Acute heart failure 3 Acute liver failure 4 Intraabdominal hemorrhage 5 Ovarian fibroma

Ovarian fibroma Explanation Ascites in a young woman are most often caused by the liver, pancreatic/renal disease, pelvic/abdominal tumors, and infection. The SAAG <1.1 g/dL points to the non-portal hypertensive ascites. This case is Meigs syndrome, characterized by the triad of benign ovarian tumor (usually fibroma), ascites, and pleural effusion that resolves after resection of the tumor. Menstrual irregularity and amenorrhea are probably caused by a fibrous growth in this patient's ovary, which causes abnormal levels of sex hormone production. An extrauterine pregnancy will not present with the ascites and pleural effusion. The clinical presentation of ectopic pregnancy occurs at approximately 2 months after the last normal menstrual period; it typically causes vaginal bleeding and abdominal pain. Acute heart failure may present with shortness of breath. Ascites and ovarian tumor may be the accidental finding, but SAAG >1.1 g/dL rules out non-portal hypertensive ascites. The SAAG is calculated by subtracting the ascitic fluid albumin value from the serum albumin value, and it correlates directly with portal pressure. It is the best single test for classifying the ascites into the ascites caused by portal hypertension (SAAG >1.1 g/dL) and non-portal hypertension (SAAG <1.1 g/dL). Its accuracy is about 97%. Acute liver failure is not likely when there are normal CBC and chemistry panels with non-portal hypertensive SAAG. Visible bloody ascites requires the presence of more than 20,000 red blood cells/µL. It is usually a consequence of either malignancy or a traumatic tap. A traumatic tap results in a heterogeneously bloody fluid that will clot. A traumatic tap is most probably the case of bloody peritoneal fluid.

Case A 49-year-old woman presents with a 1-month history of a rash on her right breast. She reports that it has not been responding to the creams that she has been applying and she is having some burning and itching at the rash. On examination, she has an erythematous area that is sharply demarcated with scaling and vesicles over her right areola and nipple. There is a nontender lump in the upper outer quadrant. Her left breast is normal. Question What is the most likely diagnosis? Answer Choices 1 Basal cell carcinoma 2 Squamous cell carcinoma 3 Malignant melanoma 4 Kaposi sarcoma 5 Paget's disease

Paget's disease Explanation In Paget's disease of the breast, patients present with a unilateral sharply marginated red scaly rash that can be vesicular or ulcerated affecting the nipple and areola. There may also be an underlying breast cancer. Treatment generally includes mastectomy if a malignancy is present. There are several types of malignant melanomas, including: Superficial spreading melanomas—macule or thin plaque with a variation in color (blue, black, grey, etc) Nodular melanomas—darkly pigmented, pedunculated, or polypoid papules or nodules with small size, uniform color, and symmetric borders Lentigo maligna melanomas—tan or brown macules with scattered darker spots and raised areas on sun-exposed areas, such as the face Acral lentiginous melanomas—most common form in African American patients; they develop on the palms, sole, subungual skin, and mucous membranes. They appear dark brown to black and are irregularly pigmented macules or patches with raised areas, ulceration, bleeding, and a larger diameter. Patients with basal cell carcinomas (BCC) can present with a shiny pearly pink or flesh-toned papule with telangiectasia (nodular BCC), or with erythematous scaling thin plaques, patches, or macules (superficial BCC), or with a firm flesh toned to pink plaque or papules that are atrophic with ill-defined borders (morpheaform BCC). Patients with squamous cell carcinomas (SCC) may present with a wide variety of clinical manifestations, including papules, plaques, or nodules, and smooth, hyperkeratotic, or ulcerative lesions. Typically they are located on sun-exposed areas, especially the head and neck (55%) and only 4% on chest and abdomen. They can invade the underlying tissue and metastasize. There are several forms of Kaposi sarcoma (KS), including: Classic KS—purple plaques or nodules on the lower limbs of elderly men of Mediterranean origin without AIDS AIDS-related form—widely disseminated purple papules or plaques on the skin, mucous membranes, and viscera Endemic form—involves lymph nodes and skin. Found in Africa. aggressive in children iatrogenic form—immunosuppressive form on distal LEs that can sometimes disseminate

Case A 75-year-old woman presents due to intense pruritis of the vulva and occasional bleeding. She is unsure if the bleeding is caused by her scratching. She has tried some OTC preparations to alleviate the itching without relief. She denies vaginal discharge or dysuria. On physical examination, you notice excoriations and some scattered lesions that look like eczema on the vulva; they do not scrape off. Also noted was inguinal lymphadenopathy. You decide to do a punch biopsy. The pathology report reveals large eosinophilic cells. Question What is the most likely diagnosis? 1 Lichen sclerosis 2 Squamous cell carcinoma 3 Paget's disease 4 Melanoma 5 Candida albicans

Paget's disease Explanation Paget's disease is associated with intense pruritus of the vulva and lesions that resemble eczema. Pathology also characteristically shows large eosinophilic Paget's cells. Lichen sclerosis pathology would reveal changes associated with chronic inflammation. A majority of vulvar carcinomas are histologically squamous cell carcinomas, but the pathology does not indicate that in this case. Melanoma lesions will generally appear hyperpigmented. Candida albicans is also not associated with inguinal lymphadenopathy. The lesions that appear on the vulva do not scrape off, and she does not have any vaginal discharge.

Case A 24-year-old woman presents with a 12-hour history of lower abdominal pain, nausea, and vomiting. She rates the pain at a 4 last night when it initially began, but she currently puts it at a 9; she states that the pain seems to be worsening with each passing hour. She states she had her menses 1 week prior, noting an irregular flow and excessive vaginal discharge since it ceased. She is single and admits to not being monogamous; she only occasionally uses barrier contraception during sexual encounters. She takes a daily oral contraceptive pill. Question Highlights What is the most likely diagnosis? Answer Choices 1 Ectopic pregnancy 2 Pelvic inflammatory disease 3 Appendicitis 4 Bacterial vaginosis 5 Endometriosis

Pelvic inflammatory disease Explanation This patient most likely has an active case of salpingitis/endometritis, commonly referred to as pelvic inflammatory disease (PID). PID is a severe gynecological infection that is often polymicrobial; Neisseria gonorrhea and Chlamydia trachomatis are the most common sexually transmitted organisms that lead to PID. PID is frequently found in young nulliparous sexually active women with multiple partners. PID presents with lower abdominal pain, chills, fever, menstrual disturbances, and purulent cervical discharge. Physical exam will reveal exquisite uterine, adnexal or cervical motion tenderness, and cervical discharge. Ectopic pregnancy is a possibility, but it is lower on the overall differential because the patient experienced menses just 1 week prior. An ectopic pregnancy is even less likely because the patient currently takes oral contraceptive pills. Appendicitis is a possibility and will commonly present with periumbilical pain (early), right lower quadrant pain and tenderness (later), anorexia, nausea, vomiting, or low-grade fever. Presence of an unusual vaginal discharge put this diagnosis lower in the differential. Bacterial vaginosis (BV) is also considered a polymicrobial disease, but it is not considered a sexually transmitted infection. BV will lead to a malodorous discharge but will not typically have any signs or symptoms or inflammation. Endometriosis is a very unpredictable disease state, variable in its presentation and course. Patients will most often report varying degrees of dysmenorrhea, chronic pelvic pain, and dyspareunia. It is not an infection, is not caused by an organism, and is not a sexually transmitted disease. This patient has minor details similar to endometriosis, but this case is not consistent with the general findings of this disease state.

Case A 58-year-old woman with no significant past medical history presents with a 6-month history of "heartburn," sometimes occurring after meals. There is associated fatigue, bloated abdominal sensation, early satiety, and alternating constipation and diarrhea. She denies fever, chills, changes in weight, chest pain, shortness of breath, abdominal pain, nausea, vomiting, melena, hematochezia, and vaginal discharge. Her last menstrual period was 4 years ago. She is unmarried, and she does not have any children. Her physical exam reveals normal vital signs and a normal cardiopulmonary exam. Her abdomen is protuberant; there is a shifting dullness and a fluid wave noted. The pelvic exam reveals a solid irregular fixed lesion in the left lower abdomen. Question What is the most appropriate initial intervention for this patient at this time? 1 Perform a transvaginal ultrasound. 2 Prescribe a proton pump inhibitor. 3 Refer the patient for an upper endoscopy. 4 Prescribe medication for irritable bowel syndrome. 5 Observation and reassessment in 1-3 months.

Perform a transvaginal ultrasound. Explanation This patient's presentation suggests ovarian cancer. Doppler transvaginal ultrasonography is used in the initial evaluation, and it is the most useful initial investigational tool in the assessment of adnexal masses. Ultrasonography may define the morphology of the pelvic tumor. In addition, it can determine whether the suspected tumor has metastasized to other abdominal organs, such as the liver. Sonographic features suggestive of cancer include complexity with solid and cystic areas, extramural fluid, echogenicity, wall thickening, septa, and papillary projections. Proton pump inhibitors and irritable bowel therapeutics should not be initiated without adequate investigation into the presence of abdominopelvic malignancy. In patients with diffuse carcinomatosis and gastrointestinal symptoms, a GI tract workup and endoscopy may be indicated as an adjunctive modality to a transvaginal sonogram.

Case A 16-year-old otherwise healthy girl presents with cessation of her menstrual cycle. She does not take any medications or use tobacco or drugs. She has a boyfriend, but she denies intercourse. Her family history is negative for gynecologic or fertility problems, autoimmune diseases, and endocrinopathies; her mother's and female relatives' menarche presented at age 12-14. There are no syndromic features on examination. Her height, weight, and BMI are within normal range, and her vital signs are normal. Her skin is clear (no acne), and there are no signs of hirsutism. There is no thyromegaly. Her breasts are developed, and pubic and axillary hair is present. Her abdomen is benign; her external genitalia maturated; there is no clitoromegaly; vaginal mucosa is moist and pink; saline-moistened applicator swab reveals normal vaginal length. Single finger examination demonstrates the presence of uterus. Neurological examination is non-focal; sense of smell is preserved; visual fields are normal by confrontation; fundoscopic examination shows no papilledema. Question What will you consider first in further evaluation of amenorrhea in this girl? Answer Choices 1 Pubertal delay 2 Chromosomal abnormality 3 Pregnancy 4 Hypothalamic amenorrhea 5 Gonadal dysgenesis

Pregnancy Explanation Pregnancy must be considered as a possible causative factor of secondary amenorrhea and is a common cause that should be tested for regardless of history. Denial of intercourse among adolescents is common, or the patient may not be able to disclose sexual activity or abuse. Around 98% of girls in the US achieve menarche by age 15. The presence of secondary sexual characteristics makes the evaluation of pubertal delay unnecessary. Chromosomal abnormalities may also present with amenorrhea (e.g., Turner syndrome, androgen insensitivity). General appearance may reveal some syndromes (e.g., Turner syndrome with webbed neck, shield chest, widely spaced nipples) and midline facial defects associated with hypothalamic-pituitary axis problems and renal and vertebral anomalies in Müllerian defects, etc. Physical examination does not suggest these diagnoses, so the more probable and affordable diagnostic approach should be considered first. Functional hypothalamic amenorrhea is a diagnosis of exclusion when pregnancy and other pathologic causative agents are not found. There are no signs of pituitary or ovarian failure or gonadal dysgenesis in this patient. As axillary and pubic hair is developed, her adrenal androgens are probably normal. The presence of developed breasts and maturation of the external genitalia, vagina, and uterus shows that estrogen levels are probably within normal limits.

Case A 25-year-old woman presents to discuss her available contraception options. You review her history and note menses onset at age 12, duration of menses typically around 6 days, occurring every 30 days. She is G0P0, and she has no history of abnormal pap smears or diagnosed STIs. The patient is a non-smoker in a monogamous relationship with one partner for the past year. All other medical history is non-contributory with the exception of the patient having a deep venous thrombosis at age 19 and a pulmonary embolism at age 21. Other than anticoagulation therapy for the appropriate amount of time, no other hematological evaluation was pursued after these events. Question Given the patient's history, what form of contraception would you select? Answer Choices 1 Estrogen/progestin combination oral contraception pill 2 Progestin-only oral contraception pill 3 Etonogestrel/ethinyl estradiol vaginal ring 4 Norelgestromin/ethinyl estradiol patch-transdermal 5 Estradiol/medroxyprogesterone monthly injection

Progestin-only oral contraception pill Explanation This patient is an overall healthy 25-year-old woman, but her history of multiple hypercoagulable events without any substantial risk factors puts her in the likely category of possessing an inherited hypercoagulable state. The most common inherited hypercoagulable state is Factor V Leiden. Although this patient is likely to have Factor V Leiden, she can still be offered a type of contraceptive method to prevent unplanned pregnancy. The common pre-thrombotic contraceptive component is estrogen. The safest method of contraception would only contain progestin: progestin-only implants, a monthly injection of only progestin, levonorgestrel-based intrauterine device, and progestin-only pills, also referred to as the "mini-pill." The estrogen/progestin combination oral contraception pill, etonogestrel/ethinyl estradiol vaginal ring, norelgestromin/ethinyl estradiol patch-transdermal, and estradiol/medroxyprogesterone monthly injection all contain a form of estrogen, which should be strictly avoided in this patient.

Case A 32-year-old nulliparous woman presents for a routine gynecological exam. She has been married for 5 years and plans to start a family in the near future. Menarche occurred at age 11. Menstrual cycles are regular, occurring every 28-30 days, and lasting for 4-5 days each. She denies menorrhagia and dysmenorrhea. She has used oral contraceptive pills since age 18; she recently discontinued them and began taking prenatal vitamins. On physical examination, you palpate a mass on the left side. A transvaginal ultrasound confirms a 3 cm complex cystic mass on the left ovary, without free fluid detected within the pelvis. The patient has no family history of any malignant tumors. Question What is the best therapeutic approach for this patient? Answer Choices 1 Resume oral contraceptive pills 2 Repeat ultrasound in 2-3 months 3 Plan an oophorectomy 4 Schedule a cystectomy 5 Perform a hysterosalpingogram

Repeat ultrasound in 2-3 months Explanation As this patient is pre-menopausal with no family history and the adnexal mass is cystic, unilateral, and asymptomatic without the presence of ascites, the possibility of the ovarian cyst being malignant is very small. Most benign cysts regress within 2 menstruation cycles, so planning a repeat ultrasound in 2-3 months is the most reasonable approach. If at that time the cyst has increased in size, a laparoscopic cystectomy may be indicated. An oophorectomy would not be necessary unless the surgeon suspected malignancy or the cyst encompassed the ovary itself. Another conservative approach would be to resume oral contraceptive pills, but this patient desires pregnancy. The 3 cm cyst is likely to resolve on its own and pregnancy is not contraindicated in this particular case. The option to perform a hysterosalpingogram is not associated with the management of an ovarian cyst. This procedure is a part of an infertility workup to assess tubal patency and uterine anatomic architecture.

Case A 28-year-old woman presents with abdominal pain and vaginal bleeding. The pain began last night as a dull ache in the right lower quadrant, but this morning it became much more severe. She reports dizziness and nausea. Her last menstrual period was 6 weeks ago. On examination, the patient is afebrile; pulse 100/min, BP 86/60 mm Hg, RR 20/min. Physical exam reveals moderate to severe tenderness in the right lower quadrant with rebound tenderness and guarding. Pelvic exam reveals a small amount of blood at the cervical os with cervical motion tenderness. A variety of laboratory tests have been ordered. Question What is the most likely diagnosis? Answer Choices 1 Acute appendicitis 2 Acute salpingitis 3 Tubo-ovarian abscess 4 Ruptured ectopic pregnancy 5 Ruptured ovarian cyst

Ruptured ectopic pregnancy Explanation The most likely diagnosis is a ruptured ectopic pregnancy, defined as extra-uterine implantation of a pregnancy, which occurs most commonly in the fallopian tube (96%). Symptoms include vaginal spotting and crampy pain as early as 4-5 weeks after the last menstrual period. The tubal pregnancy will usually bleed slowly at first, but when the pregnancy expands to the point of tubal rupture, patients can present with rapid hemorrhage, shock, and then death. On exam, tenderness and signs of peritoneal irritation are present. The uterus will be enlarged, but smaller than expected for age of pregnancy. There is often cervical motion tenderness, and an adnexal mass may be felt. Abdominal pain with vaginal bleeding accompanied by hypotension, marked abdominal guarding, and rebound tenderness is suggestive of ruptured ectopic pregnancy. Appendicitis is most likely to present with abdominal pain (classically starts periumbilical and then migrates to right lower quadrant), nausea/vomiting, bloating/gas, diarrhea, and malaise. Exam may be subtle at first, leading to RLQ tenderness, high-grade fever, McBurney's point tenderness, Rovsing's sign, psoas sign, and obturator sign. Salpingitis (pelvic inflammatory disease) is an infection of the upper genital tract. Patients present with acute onset of pelvic/lower abdominal pain that worsens with jarring movement. Vaginal discharge is common, but vaginal bleeding only occurs in 1/3 of patients. Ruptured ovarian cyst presents with an acute onset of abdominal pain, most often occurring during physical activity classically occurring mid-cycle. Additional symptoms include vaginal bleeding, weakness, nausea, or vomiting. Physical exam reveals lower abdominal pain, low-grade fever, and an adnexal mass may be palpable. Peritoneal signs may be present. Hypotension may occur with significant hemorrhage. Tubo-ovarian abscess (TOA) is a complication of pelvic inflammatory disease. Patients present with acute lower abdominal pain, vaginal discharge, fever/chills. Patients with a ruptured TOA present with sepsis and adnexal mass.

Case A 36-year-old G1P0010 woman and her 40-year-old husband present for a family planning visit as they have been trying to conceive for 10 months without results. She has history of one spontaneous abortion five years ago. The spouse has never fathered a child. Neither partner has any other significant past medical or past surgical history. Physical examination of both partners is unremarkable. Question What do you advise as the next step in the couple's evaluation? 1 Hysterosalpingogram 2 Intrauterine insemination 3 Laparoscopy 4 Semen analysis 5 Thrombophilia panel

Semen analysis Explanation In general, infertility is defined as not being able to conceive after one year of unprotected sex. However, women who are older than 35 years and have not conceived after a 6-month period of trying to do so should be evaluated for infertility; while women over 40 years old should be evaluated immediately. Your patient had one pregnancy; therefore, she is probably able to conceive. In approximately 35% of couples, the male factor is responsible for infertility. The next step would be to perform semen analysis and evaluate the motility, ejaculate volume, concentration, and the morphology of sperm. Laparoscopy is an invasive procedure that should only be considered after other non-invasive structural evaluations of infertility. Hysterosalpingogram (HSG) is a good choice to evaluate tubal patency; however, it is also an invasive procedure and is not one of the first steps for an infertility workup. Intrauterine insemination (IUI) is an option for patients with male and/or male and female infertility. A semen analysis and an HSG would be done prior to an IUI. A thrombophilia panel is appropriate as part of the work-up for a habitual aborter, which is defined as three or more consecutive spontaneous or missed abortions. Other lab work would be appropriate for a routine fertility work-up - hormones, antimullerian hormone, ovulation evaluation; in addition, pelvic ultrasound is recommended.

Case A 40-year-old Caucasian man presents to your office to ask for advice regarding the inability of his wife to conceive. She is younger than him by 8 years and he believes she is in good health. Additionally, she has two children from a previous marriage, both pregnancies without complication. He was also married once before, but his first wife did not conceive children. He has not yet undergone any fertility tests. Question What is the most common cause for infertility in this case? Answer Choices 1 Endometrial cavity and shape 2 Spermatogenesis and motility 3 Endocrine and liver dysfunction 4 Mucus and sperm interaction 5 Size of genitalia

Spermatogenesis and motility Explanation Spermatogenesis and sperm motility need to be evaluated, as this is the most likely cause in male factor infertility. Sperm analysis is key in the initial evaluation of male factor infertility. External genital abnormalities, including size of genitalia and incomplete sexual development, can contribute to infertility, but it is not the most common cause. His partner had two previous successful pregnancies, indicating her endometrial cavity and shape is not a contributing factor. Mucus and sperm interaction and labs to rule out endocrine and liver dysfunction should be considered if spermatogenesis and motility are normal.

Case A 36-year-old woman gravida 1 para 1 came to the emergency department with severe abdominal pain and two episodes of profuse vaginal bleeding, chills, and light-headedness 10 days after cesarean delivery. Because of her age, she was closely monitored during the pregnancy and several ultrasound examinations were performed. Today, physical examination reveals tender and firm abdomen with bloody vaginal discharge; her pulse is 100/min; blood pressure of 100/60 mm Hg; and temperature 36.0°C. Laboratory reveals a white blood cell count of 10,000/μL and hemoglobin level of 11 g/DL. Coagulation tests and urine beta hCG are within normal limits. Ultrasound reveals low-resistance vessel in the inner third of the myometrium. Question What is the most probable cause of her bleeding? 1 Subinvolution of the placental site 2 Gestational trophoblastic disease 3 Endometritis 4 Disseminated intravascular coagulation 5 Leiomyoma uteri

Subinvolution of the placental site Explanation This patient has a late postpartum hemorrhage, and ultrasound shows increased myometrial vascularity; these findings are most commonly associated with retained products of conception, uterine arteriovenous malformations, or subinvolution of the placental bed. Normal beta hCG excludes gestational trophoblastic disease. Endometritis is unlikely in an afebrile patient without other signs of infection. You should think about the disseminated intravascular coagulation in a patient with severe generalized hemorrhaging and laboratory findings of elevated prothrombin, PT, APTT, reduced fibrinogen and platelets, and high levels of fibrin degradation products. Her coagulation tests are normal. Leiomyomas are benign smooth muscle neoplasms. Your patient had several ultrasound examinations and a cesarean section. None of them revealed the presence of leiomyoma, so a low-resistance vessel in the inner third of the myometrium is most probably a post-delivery event.

Case A 65-year-old woman presents with bloating, early satiety, changes in her bowel habits, and an unintentional 10 lb weight gain over 3 months. Upon further questioning, you discover that she never had children and her sister passed away 20 years ago from some sort of cancer in her belly. On her physical exam, an adnexal mass is felt. Question What is the most appropriate next step to further evaluate this patient? 1 CA125 2 Colonoscopy 3 Genetic testing 4 Mammogram 5 Transvaginal ultrasound

Transvaginal ultrasound Explanation A transvaginal ultrasound will help distinguish the characteristics of the adnexal mass, cystic formations, septations within the mass, and the presence of ascites. Excluding malignancy is the priority in a postmenopausal patient presenting with these symptoms. The tumor marker CA125 is used in the setting of a patient with ovarian cancer. This diagnosis should be suspected in this patient, but it is not a diagnostic test for ovarian cancer. It is more warranted before and during chemotherapy. CA125 can be elevated in several disease processes to include colon, breast, hepatic, and uterine. It can also be within normal limits (<35 IU/mL) in early stages of ovarian cancer. A colonoscopy would evaluate the bowel further but would not obtain the characteristics of the adnexal mass felt on exam. Genetic testing is pursued after a cancer diagnosis is made. It is not used during the initial evaluation of the mass. A mammogram can be obtained in the workup to reveal the possibility of metastatic breast cancer or simultaneous breast cancer.

Case A 16-year-old sexually active girl is seen for a 2-month history of amenorrhea. She denies unprotected sex but relies on her partner to use a condom. She has vomited in the early morning twice this week and had vaginal spotting for 3 days accompanied by cramping lower abdominal pain that became sharp. Onset of menses was at age 12, with normal regular periods since then. No history of sexually transmitted disease. Normal vital signs. Slight right and left lower quadrant abdominal tenderness without guarding and rebound is present. Cervix is closed. No blood is seen in the vaginal vault. The uterus is not palpable. Serum β-HCG: 5200 mIU/mL. Vaginal spotting has increased, and abdominal pain has become more frequent. Repeat examination 3 days later is unchanged. The uterus is still not palpable. Repeat serum β-HCG is 6800 mIU/mL. Transvaginal ultrasound fails to reveal an intrauterine pregnancy or gestational sac. Question What is the most likely diagnosis? Answer Choices 1 Cervical ectopic pregnancy 2 Choriocarcinoma 3 Pseudocyesis 4 Tubal ectopic pregnancy 5 Early intrauterine gestation

Tubal ectopic pregnancy Explanation The absence of fetal pole or gestational sac in the uterus with elevated serum β-HCG indicates an ectopic pregnancy. Approximately 96% of ectopic pregnancies occur in the fallopian tube (tubal pregnancy). Cervical ectopic pregnancies are rare, so this is a less likely diagnosis. The full triad of abdominal/pelvic pain, amenorrhea, and irregular vaginal bleeding is present in only half of patients. Abdominal/pelvic pain may be unilateral or bilateral, usually worse on the affected side. Choriocarcinoma is a malignant neoplasm secondary to trophoblastic proliferation. Patients with choriocarcinoma have significantly elevated β-HCG levels, sometimes as high as 100,000 mIU/mL. This patient's β-HCG is not adequately elevated to suggest choriocarcinoma. Serum β-HCG is not elevated in pseudocyesis (false pregnancy), a condition in which the female patient truly believes she is pregnant and may exhibit signs and symptoms of true pregnancy such as amenorrhea, hyperemesis, breast swelling and tenderness, weight gain, abdominal tenderness, and even quickening. Transvaginal ultrasound failed to reveal a fetal pole or gestational sac in the uterus, which should be detected by 36-40 days in the case of early intrauterine gestation.

Case A 32-year-old G2P2 woman presents 1 month post-intrauterine contraceptive device placement for follow-up position check. The patient states that she can no longer feel the strings from the device. She was told to check occasionally to make sure it stayed positioned properly. Upon vaginal exam, you confirm that the strings are no longer visible in the cervical os. Question What is the most appropriate procedure to evaluate this patient? Answer Choices 1 CT scan 2 Laparoscopy 3 MRI 4 Ultrasound 5 X-ray

Ultrasound Explanation The fastest and best method to determine the location of an IUD is an ultrasound of the uterus. This can locate the IUD even if it has eroded through the lining of the uterus. This method is inexpensive, typically available in clinic, and does not expose the patient to radiation. If the IUD has perforated the uterus, then a laparoscopy may be necessary to remove the device. Other tests are not usually performed in this situation.


Kaugnay na mga set ng pag-aaral

Service Cloud Certification Prep

View Set

Chapter 4 - Videos ( extra credit)

View Set

7 ARTICLES OF THE US CONSTITUTION

View Set

The Art of Public Speaking - Chapter 9

View Set